SAT Exam  >  SAT Tests  >  Digital SAT Mock Test Series 2024  >  OneTime: Digital SAT Mock Test - 5 - SAT MCQ

OneTime: Digital SAT Mock Test - 5 - SAT MCQ


Test Description

30 Questions MCQ Test Digital SAT Mock Test Series 2024 - OneTime: Digital SAT Mock Test - 5

OneTime: Digital SAT Mock Test - 5 for SAT 2024 is part of Digital SAT Mock Test Series 2024 preparation. The OneTime: Digital SAT Mock Test - 5 questions and answers have been prepared according to the SAT exam syllabus.The OneTime: Digital SAT Mock Test - 5 MCQs are made for SAT 2024 Exam. Find important definitions, questions, notes, meanings, examples, exercises, MCQs and online tests for OneTime: Digital SAT Mock Test - 5 below.
Solutions of OneTime: Digital SAT Mock Test - 5 questions in English are available as part of our Digital SAT Mock Test Series 2024 for SAT & OneTime: Digital SAT Mock Test - 5 solutions in Hindi for Digital SAT Mock Test Series 2024 course. Download more important topics, notes, lectures and mock test series for SAT Exam by signing up for free. Attempt OneTime: Digital SAT Mock Test - 5 | 154 questions in 180 minutes | Mock test for SAT preparation | Free important questions MCQ to study Digital SAT Mock Test Series 2024 for SAT Exam | Download free PDF with solutions
OneTime: Digital SAT Mock Test - 5 - Question 1

Question based on the following passage.
This passage is from Ralph Waldo Emerson, "Prudence." Public domain. First published in 1841.
What right have I to write on prudence, of
which I have little, and that of the negative sort?
My prudence consists in avoiding and going
without, not in the inventing of means and
(5) methods, not in adroit steering, not in gentle
repairing. I have no skill to make money spend
well, no genius in my economy, and whoever sees
my garden discovers that I must have some other
garden. Yet I love facts, and hate shiftiness and
(10) people without perception.
Then I have the same title to write on
prudence that I have to write on poetry or
holiness. We write from aspiration as well as from
experience.
(15) We paint those qualities that we do not
possess. The poet admires the man of energy
and tactics; the merchant breeds his son for the
church or the bar; and where a man is not vain and
egotistic you shall find what he lacks, by his praise.
(20) Yet it would be hardly honest for me not
to balance these fine lyric words with words
of coarser sound. Prudence is the virtue of the
senses. It is the science of appearances. It is the
outmost action of the inward life. It is God taking
(25) thought for oxen. It moves matter after the laws
of matter. It is content to seek health of body by
complying with physical conditions, and health
of mind by the laws of the intellect.
The world of the senses is a world of shows;
(30) it does not exist for itself, but has a symbolic
character; and a true prudence or law of shows
recognizes the co-presence of other laws and
knows that its own office is secondary; knows
that it is surface and not center where it works.
(35) Prudence is false when detached. It is legitimate
when it is the natural history of the soul
incarnate, when it unfolds the beauty of laws
within the narrow scope of the senses.
There are all degrees of proficiency in
(40) knowledge of the world. It is sufficient to our
present purpose to indicate three. One class lives
to the utility of the symbol, esteeming health and
wealth a final good. Another class lives above this
mark, to the beauty of the symbol, as the poet and
(45) artist and the naturalist and man of science.
A third class lives above the beauty of the symbol
to the beauty of the thing signified; these are
wise men. The first class has common sense; the
second, taste; and the third, spiritual perception.
(50) Once in a long time, a man traverses the whole
scale, and sees and enjoys the symbol solidly, then
also has a clear eye for its beauty, and lastly, while
he pitches his tent on this sacred volcanic isle of
nature, does not offer to build houses and barns
(55) thereon, reverencing the splendor of the God
which he sees bursting through each chink and
cranny.
The world is filled with the proverbs and
acts of a base prudence, which is a devotion to
(60) matter, as if we possessed no other faculties than
the palate, the nose, the touch, the eye and ear; a
prudence that never subscribes, that never gives,
that seldom lends, and asks but one question of
any project: will it bake bread? This is a disease
(65) like a thickening of the skin until the vital organs
are destroyed. But culture, revealing the high
origin of the apparent world and aiming at the
perfection of the man as the end, degrades every
thing else, as health and bodily life, into means.
(70) It sees prudence not to be a separate faculty,
but a name for wisdom and virtue conversing
with the body and its wants. Cultivated men
always feel and speak so, as if a great fortune, the
achievement of a civil or social measure, great
(75) personal influence, a graceful and commanding
address, had their value as proofs of the energy
of the spirit. If a man loses his balance and
immerses himself in any trades or pleasures for
their own sake, he may be a good wheel or pin,
(80) but he is not a cultivated man.

Q. The tone of the first paragraph is best described as

Detailed Solution for OneTime: Digital SAT Mock Test - 5 - Question 1

In the first paragraph, the author tells us that he has little prudence and no skill in inventing of means and methods . . . in adroit steering. . . nor in gentle repairing. He also has no skill to make money spend well. These are self-effacing descriptions. They are certainly not pontifical (speaking as a high priest), aspirational (expressing high hopes and goals), or sardonic (grimly cynical).

OneTime: Digital SAT Mock Test - 5 - Question 2

Question based on the following passage.
This passage is from Ralph Waldo Emerson, "Prudence." Public domain. First published in 1841.
What right have I to write on prudence, of
which I have little, and that of the negative sort?
My prudence consists in avoiding and going
without, not in the inventing of means and
(5) methods, not in adroit steering, not in gentle
repairing. I have no skill to make money spend
well, no genius in my economy, and whoever sees
my garden discovers that I must have some other
garden. Yet I love facts, and hate shiftiness and
(10) people without perception.
Then I have the same title to write on
prudence that I have to write on poetry or
holiness. We write from aspiration as well as from
experience.
(15) We paint those qualities that we do not
possess. The poet admires the man of energy
and tactics; the merchant breeds his son for the
church or the bar; and where a man is not vain and
egotistic you shall find what he lacks, by his praise.
(20) Yet it would be hardly honest for me not
to balance these fine lyric words with words
of coarser sound. Prudence is the virtue of the
senses. It is the science of appearances. It is the
outmost action of the inward life. It is God taking
(25) thought for oxen. It moves matter after the laws
of matter. It is content to seek health of body by
complying with physical conditions, and health
of mind by the laws of the intellect.
The world of the senses is a world of shows;
(30) it does not exist for itself, but has a symbolic
character; and a true prudence or law of shows
recognizes the co-presence of other laws and
knows that its own office is secondary; knows
that it is surface and not center where it works.
(35) Prudence is false when detached. It is legitimate
when it is the natural history of the soul
incarnate, when it unfolds the beauty of laws
within the narrow scope of the senses.
There are all degrees of proficiency in
(40) knowledge of the world. It is sufficient to our
present purpose to indicate three. One class lives
to the utility of the symbol, esteeming health and
wealth a final good. Another class lives above this
mark, to the beauty of the symbol, as the poet and
(45) artist and the naturalist and man of science.
A third class lives above the beauty of the symbol
to the beauty of the thing signified; these are
wise men. The first class has common sense; the
second, taste; and the third, spiritual perception.
(50) Once in a long time, a man traverses the whole
scale, and sees and enjoys the symbol solidly, then
also has a clear eye for its beauty, and lastly, while
he pitches his tent on this sacred volcanic isle of
nature, does not offer to build houses and barns
(55) thereon, reverencing the splendor of the God
which he sees bursting through each chink and
cranny.
The world is filled with the proverbs and
acts of a base prudence, which is a devotion to
(60) matter, as if we possessed no other faculties than
the palate, the nose, the touch, the eye and ear; a
prudence that never subscribes, that never gives,
that seldom lends, and asks but one question of
any project: will it bake bread? This is a disease
(65) like a thickening of the skin until the vital organs
are destroyed. But culture, revealing the high
origin of the apparent world and aiming at the
perfection of the man as the end, degrades every
thing else, as health and bodily life, into means.
(70) It sees prudence not to be a separate faculty,
but a name for wisdom and virtue conversing
with the body and its wants. Cultivated men
always feel and speak so, as if a great fortune, the
achievement of a civil or social measure, great
(75) personal influence, a graceful and commanding
address, had their value as proofs of the energy
of the spirit. If a man loses his balance and
immerses himself in any trades or pleasures for
their own sake, he may be a good wheel or pin,
(80) but he is not a cultivated man.

Q. The author’s reference to “some other garden” (lines 8–9) primarily suggests that he

Detailed Solution for OneTime: Digital SAT Mock Test - 5 - Question 2

The statement that whoever sees my garden discovers that I must have some other garden is the last statement in the author's list of his personal inadequacies. Therefore, this statement must be taken to be self-effacing as the other statements are, and specifically to mean that he lacks gardening skill.

1 Crore+ students have signed up on EduRev. Have you? Download the App
OneTime: Digital SAT Mock Test - 5 - Question 3

Question based on the following passage.
This passage is from Ralph Waldo Emerson, "Prudence." Public domain. First published in 1841.
What right have I to write on prudence, of
which I have little, and that of the negative sort?
My prudence consists in avoiding and going
without, not in the inventing of means and
(5) methods, not in adroit steering, not in gentle
repairing. I have no skill to make money spend
well, no genius in my economy, and whoever sees
my garden discovers that I must have some other
garden. Yet I love facts, and hate shiftiness and
(10) people without perception.
Then I have the same title to write on
prudence that I have to write on poetry or
holiness. We write from aspiration as well as from
experience.
(15) We paint those qualities that we do not
possess. The poet admires the man of energy
and tactics; the merchant breeds his son for the
church or the bar; and where a man is not vain and
egotistic you shall find what he lacks, by his praise.
(20) Yet it would be hardly honest for me not
to balance these fine lyric words with words
of coarser sound. Prudence is the virtue of the
senses. It is the science of appearances. It is the
outmost action of the inward life. It is God taking
(25) thought for oxen. It moves matter after the laws
of matter. It is content to seek health of body by
complying with physical conditions, and health
of mind by the laws of the intellect.
The world of the senses is a world of shows;
(30) it does not exist for itself, but has a symbolic
character; and a true prudence or law of shows
recognizes the co-presence of other laws and
knows that its own office is secondary; knows
that it is surface and not center where it works.
(35) Prudence is false when detached. It is legitimate
when it is the natural history of the soul
incarnate, when it unfolds the beauty of laws
within the narrow scope of the senses.
There are all degrees of proficiency in
(40) knowledge of the world. It is sufficient to our
present purpose to indicate three. One class lives
to the utility of the symbol, esteeming health and
wealth a final good. Another class lives above this
mark, to the beauty of the symbol, as the poet and
(45) artist and the naturalist and man of science.
A third class lives above the beauty of the symbol
to the beauty of the thing signified; these are
wise men. The first class has common sense; the
second, taste; and the third, spiritual perception.
(50) Once in a long time, a man traverses the whole
scale, and sees and enjoys the symbol solidly, then
also has a clear eye for its beauty, and lastly, while
he pitches his tent on this sacred volcanic isle of
nature, does not offer to build houses and barns
(55) thereon, reverencing the splendor of the God
which he sees bursting through each chink and
cranny.
The world is filled with the proverbs and
acts of a base prudence, which is a devotion to
(60) matter, as if we possessed no other faculties than
the palate, the nose, the touch, the eye and ear; a
prudence that never subscribes, that never gives,
that seldom lends, and asks but one question of
any project: will it bake bread? This is a disease
(65) like a thickening of the skin until the vital organs
are destroyed. But culture, revealing the high
origin of the apparent world and aiming at the
perfection of the man as the end, degrades every
thing else, as health and bodily life, into means.
(70) It sees prudence not to be a separate faculty,
but a name for wisdom and virtue conversing
with the body and its wants. Cultivated men
always feel and speak so, as if a great fortune, the
achievement of a civil or social measure, great
(75) personal influence, a graceful and commanding
address, had their value as proofs of the energy
of the spirit. If a man loses his balance and
immerses himself in any trades or pleasures for
their own sake, he may be a good wheel or pin,
(80) but he is not a cultivated man.

Q. In line 11, “title” most nearly means

Detailed Solution for OneTime: Digital SAT Mock Test - 5 - Question 3

Recall that the first paragraph begins with the question What right have I to write on prudence. . . ? The second provides a response to this question about his right: in saying I have the same title to write on prudence as I have to write on poetry or holiness, then, he is clearly saying that he has the standing or authority to write on prudence.

OneTime: Digital SAT Mock Test - 5 - Question 4

Question based on the following passage.
This passage is from Ralph Waldo Emerson, "Prudence." Public domain. First published in 1841.
What right have I to write on prudence, of
which I have little, and that of the negative sort?
My prudence consists in avoiding and going
without, not in the inventing of means and
(5) methods, not in adroit steering, not in gentle
repairing. I have no skill to make money spend
well, no genius in my economy, and whoever sees
my garden discovers that I must have some other
garden. Yet I love facts, and hate shiftiness and
(10) people without perception.
Then I have the same title to write on
prudence that I have to write on poetry or
holiness. We write from aspiration as well as from
experience.
(15) We paint those qualities that we do not
possess. The poet admires the man of energy
and tactics; the merchant breeds his son for the
church or the bar; and where a man is not vain and
egotistic you shall find what he lacks, by his praise.
(20) Yet it would be hardly honest for me not
to balance these fine lyric words with words
of coarser sound. Prudence is the virtue of the
senses. It is the science of appearances. It is the
outmost action of the inward life. It is God taking
(25) thought for oxen. It moves matter after the laws
of matter. It is content to seek health of body by
complying with physical conditions, and health
of mind by the laws of the intellect.
The world of the senses is a world of shows;
(30) it does not exist for itself, but has a symbolic
character; and a true prudence or law of shows
recognizes the co-presence of other laws and
knows that its own office is secondary; knows
that it is surface and not center where it works.
(35) Prudence is false when detached. It is legitimate
when it is the natural history of the soul
incarnate, when it unfolds the beauty of laws
within the narrow scope of the senses.
There are all degrees of proficiency in
(40) knowledge of the world. It is sufficient to our
present purpose to indicate three. One class lives
to the utility of the symbol, esteeming health and
wealth a final good. Another class lives above this
mark, to the beauty of the symbol, as the poet and
(45) artist and the naturalist and man of science.
A third class lives above the beauty of the symbol
to the beauty of the thing signified; these are
wise men. The first class has common sense; the
second, taste; and the third, spiritual perception.
(50) Once in a long time, a man traverses the whole
scale, and sees and enjoys the symbol solidly, then
also has a clear eye for its beauty, and lastly, while
he pitches his tent on this sacred volcanic isle of
nature, does not offer to build houses and barns
(55) thereon, reverencing the splendor of the God
which he sees bursting through each chink and
cranny.
The world is filled with the proverbs and
acts of a base prudence, which is a devotion to
(60) matter, as if we possessed no other faculties than
the palate, the nose, the touch, the eye and ear; a
prudence that never subscribes, that never gives,
that seldom lends, and asks but one question of
any project: will it bake bread? This is a disease
(65) like a thickening of the skin until the vital organs
are destroyed. But culture, revealing the high
origin of the apparent world and aiming at the
perfection of the man as the end, degrades every
thing else, as health and bodily life, into means.
(70) It sees prudence not to be a separate faculty,
but a name for wisdom and virtue conversing
with the body and its wants. Cultivated men
always feel and speak so, as if a great fortune, the
achievement of a civil or social measure, great
(75) personal influence, a graceful and commanding
address, had their value as proofs of the energy
of the spirit. If a man loses his balance and
immerses himself in any trades or pleasures for
their own sake, he may be a good wheel or pin,
(80) but he is not a cultivated man.

Q. The author believes that he is justified in acting as an authority on prudence primarily because of his

Detailed Solution for OneTime: Digital SAT Mock Test - 5 - Question 4

In following his declaration that he has the right to write on prudence (lines 13-14), Emerson states that [w]e write from aspiration as well as from experience. In other words, we gain the standing to write on prudence not only from expertise in prudent behavior, but also from a focused yearning.

OneTime: Digital SAT Mock Test - 5 - Question 5

Question based on the following passage.
This passage is from Ralph Waldo Emerson, "Prudence." Public domain. First published in 1841.
What right have I to write on prudence, of
which I have little, and that of the negative sort?
My prudence consists in avoiding and going
without, not in the inventing of means and
(5) methods, not in adroit steering, not in gentle
repairing. I have no skill to make money spend
well, no genius in my economy, and whoever sees
my garden discovers that I must have some other
garden. Yet I love facts, and hate shiftiness and
(10) people without perception.
Then I have the same title to write on
prudence that I have to write on poetry or
holiness. We write from aspiration as well as from
experience.
(15) We paint those qualities that we do not
possess. The poet admires the man of energy
and tactics; the merchant breeds his son for the
church or the bar; and where a man is not vain and
egotistic you shall find what he lacks, by his praise.
(20) Yet it would be hardly honest for me not
to balance these fine lyric words with words
of coarser sound. Prudence is the virtue of the
senses. It is the science of appearances. It is the
outmost action of the inward life. It is God taking
(25) thought for oxen. It moves matter after the laws
of matter. It is content to seek health of body by
complying with physical conditions, and health
of mind by the laws of the intellect.
The world of the senses is a world of shows;
(30) it does not exist for itself, but has a symbolic
character; and a true prudence or law of shows
recognizes the co-presence of other laws and
knows that its own office is secondary; knows
that it is surface and not center where it works.
(35) Prudence is false when detached. It is legitimate
when it is the natural history of the soul
incarnate, when it unfolds the beauty of laws
within the narrow scope of the senses.
There are all degrees of proficiency in
(40) knowledge of the world. It is sufficient to our
present purpose to indicate three. One class lives
to the utility of the symbol, esteeming health and
wealth a final good. Another class lives above this
mark, to the beauty of the symbol, as the poet and
(45) artist and the naturalist and man of science.
A third class lives above the beauty of the symbol
to the beauty of the thing signified; these are
wise men. The first class has common sense; the
second, taste; and the third, spiritual perception.
(50) Once in a long time, a man traverses the whole
scale, and sees and enjoys the symbol solidly, then
also has a clear eye for its beauty, and lastly, while
he pitches his tent on this sacred volcanic isle of
nature, does not offer to build houses and barns
(55) thereon, reverencing the splendor of the God
which he sees bursting through each chink and
cranny.
The world is filled with the proverbs and
acts of a base prudence, which is a devotion to
(60) matter, as if we possessed no other faculties than
the palate, the nose, the touch, the eye and ear; a
prudence that never subscribes, that never gives,
that seldom lends, and asks but one question of
any project: will it bake bread? This is a disease
(65) like a thickening of the skin until the vital organs
are destroyed. But culture, revealing the high
origin of the apparent world and aiming at the
perfection of the man as the end, degrades every
thing else, as health and bodily life, into means.
(70) It sees prudence not to be a separate faculty,
but a name for wisdom and virtue conversing
with the body and its wants. Cultivated men
always feel and speak so, as if a great fortune, the
achievement of a civil or social measure, great
(75) personal influence, a graceful and commanding
address, had their value as proofs of the energy
of the spirit. If a man loses his balance and
immerses himself in any trades or pleasures for
their own sake, he may be a good wheel or pin,
(80) but he is not a cultivated man.

Q. Which choice provides the strongest evidence for the answer to the previous question?

Detailed Solution for OneTime: Digital SAT Mock Test - 5 - Question 5

As the explanation to the previous question indicates, the best support for this answer is in lines 13-14.

OneTime: Digital SAT Mock Test - 5 - Question 6

Question based on the following passage.
This passage is from Ralph Waldo Emerson, "Prudence." Public domain. First published in 1841.
What right have I to write on prudence, of
which I have little, and that of the negative sort?
My prudence consists in avoiding and going
without, not in the inventing of means and
(5) methods, not in adroit steering, not in gentle
repairing. I have no skill to make money spend
well, no genius in my economy, and whoever sees
my garden discovers that I must have some other
garden. Yet I love facts, and hate shiftiness and
(10) people without perception.
Then I have the same title to write on
prudence that I have to write on poetry or
holiness. We write from aspiration as well as from
experience.
(15) We paint those qualities that we do not
possess. The poet admires the man of energy
and tactics; the merchant breeds his son for the
church or the bar; and where a man is not vain and
egotistic you shall find what he lacks, by his praise.
(20) Yet it would be hardly honest for me not
to balance these fine lyric words with words
of coarser sound. Prudence is the virtue of the
senses. It is the science of appearances. It is the
outmost action of the inward life. It is God taking
(25) thought for oxen. It moves matter after the laws
of matter. It is content to seek health of body by
complying with physical conditions, and health
of mind by the laws of the intellect.
The world of the senses is a world of shows;
(30) it does not exist for itself, but has a symbolic
character; and a true prudence or law of shows
recognizes the co-presence of other laws and
knows that its own office is secondary; knows
that it is surface and not center where it works.
(35) Prudence is false when detached. It is legitimate
when it is the natural history of the soul
incarnate, when it unfolds the beauty of laws
within the narrow scope of the senses.
There are all degrees of proficiency in
(40) knowledge of the world. It is sufficient to our
present purpose to indicate three. One class lives
to the utility of the symbol, esteeming health and
wealth a final good. Another class lives above this
mark, to the beauty of the symbol, as the poet and
(45) artist and the naturalist and man of science.
A third class lives above the beauty of the symbol
to the beauty of the thing signified; these are
wise men. The first class has common sense; the
second, taste; and the third, spiritual perception.
(50) Once in a long time, a man traverses the whole
scale, and sees and enjoys the symbol solidly, then
also has a clear eye for its beauty, and lastly, while
he pitches his tent on this sacred volcanic isle of
nature, does not offer to build houses and barns
(55) thereon, reverencing the splendor of the God
which he sees bursting through each chink and
cranny.
The world is filled with the proverbs and
acts of a base prudence, which is a devotion to
(60) matter, as if we possessed no other faculties than
the palate, the nose, the touch, the eye and ear; a
prudence that never subscribes, that never gives,
that seldom lends, and asks but one question of
any project: will it bake bread? This is a disease
(65) like a thickening of the skin until the vital organs
are destroyed. But culture, revealing the high
origin of the apparent world and aiming at the
perfection of the man as the end, degrades every
thing else, as health and bodily life, into means.
(70) It sees prudence not to be a separate faculty,
but a name for wisdom and virtue conversing
with the body and its wants. Cultivated men
always feel and speak so, as if a great fortune, the
achievement of a civil or social measure, great
(75) personal influence, a graceful and commanding
address, had their value as proofs of the energy
of the spirit. If a man loses his balance and
immerses himself in any trades or pleasures for
their own sake, he may be a good wheel or pin,
(80) but he is not a cultivated man.

Q. The passage suggests that members of the “third class” (line 46) are superior for their ability to

Detailed Solution for OneTime: Digital SAT Mock Test - 5 - Question 6

The sixth paragraph (lines 39-57) discusses three classes of people according to their proficiency in knowledge of the world (lines 39-40). The first class values health and wealth [as] a final good (lines 42-43). The second class values the beauty of the symbol (line 46-47). The third class lives above the beauty of the symbol to the beauty of the thing signified (lines 46-47). This last group has spiritual perception (line 49). Therefore, its members are superior for their ability to discern sublime qualities.

OneTime: Digital SAT Mock Test - 5 - Question 7

Question based on the following passage.
This passage is from Ralph Waldo Emerson, "Prudence." Public domain. First published in 1841.
What right have I to write on prudence, of
which I have little, and that of the negative sort?
My prudence consists in avoiding and going
without, not in the inventing of means and
(5) methods, not in adroit steering, not in gentle
repairing. I have no skill to make money spend
well, no genius in my economy, and whoever sees
my garden discovers that I must have some other
garden. Yet I love facts, and hate shiftiness and
(10) people without perception.
Then I have the same title to write on
prudence that I have to write on poetry or
holiness. We write from aspiration as well as from
experience.
(15) We paint those qualities that we do not
possess. The poet admires the man of energy
and tactics; the merchant breeds his son for the
church or the bar; and where a man is not vain and
egotistic you shall find what he lacks, by his praise.
(20) Yet it would be hardly honest for me not
to balance these fine lyric words with words
of coarser sound. Prudence is the virtue of the
senses. It is the science of appearances. It is the
outmost action of the inward life. It is God taking
(25) thought for oxen. It moves matter after the laws
of matter. It is content to seek health of body by
complying with physical conditions, and health
of mind by the laws of the intellect.
The world of the senses is a world of shows;
(30) it does not exist for itself, but has a symbolic
character; and a true prudence or law of shows
recognizes the co-presence of other laws and
knows that its own office is secondary; knows
that it is surface and not center where it works.
(35) Prudence is false when detached. It is legitimate
when it is the natural history of the soul
incarnate, when it unfolds the beauty of laws
within the narrow scope of the senses.
There are all degrees of proficiency in
(40) knowledge of the world. It is sufficient to our
present purpose to indicate three. One class lives
to the utility of the symbol, esteeming health and
wealth a final good. Another class lives above this
mark, to the beauty of the symbol, as the poet and
(45) artist and the naturalist and man of science.
A third class lives above the beauty of the symbol
to the beauty of the thing signified; these are
wise men. The first class has common sense; the
second, taste; and the third, spiritual perception.
(50) Once in a long time, a man traverses the whole
scale, and sees and enjoys the symbol solidly, then
also has a clear eye for its beauty, and lastly, while
he pitches his tent on this sacred volcanic isle of
nature, does not offer to build houses and barns
(55) thereon, reverencing the splendor of the God
which he sees bursting through each chink and
cranny.
The world is filled with the proverbs and
acts of a base prudence, which is a devotion to
(60) matter, as if we possessed no other faculties than
the palate, the nose, the touch, the eye and ear; a
prudence that never subscribes, that never gives,
that seldom lends, and asks but one question of
any project: will it bake bread? This is a disease
(65) like a thickening of the skin until the vital organs
are destroyed. But culture, revealing the high
origin of the apparent world and aiming at the
perfection of the man as the end, degrades every
thing else, as health and bodily life, into means.
(70) It sees prudence not to be a separate faculty,
but a name for wisdom and virtue conversing
with the body and its wants. Cultivated men
always feel and speak so, as if a great fortune, the
achievement of a civil or social measure, great
(75) personal influence, a graceful and commanding
address, had their value as proofs of the energy
of the spirit. If a man loses his balance and
immerses himself in any trades or pleasures for
their own sake, he may be a good wheel or pin,
(80) but he is not a cultivated man.

Q. The “houses and barns” (line 54) represent

Detailed Solution for OneTime: Digital SAT Mock Test - 5 - Question 7

This phrase appears in a discussion of the individual who traverses the whole scale (line 50-51), that is, who has the skills of all three classes: practicality, taste, and spiritual perception. In saying that such a person does not offer to build houses and barns (lines 54) on the sacred volcanic isle of nature (lines 53-54), Emerson is saying that nature is merely a symbol that points to the splendor of God (55), and therefore not what a truly wise person chooses to fix his or her gaze upon. In other words, the building of houses and barns is an unwise allegiance to worldly things.

OneTime: Digital SAT Mock Test - 5 - Question 8

Question based on the following passage.
This passage is from Ralph Waldo Emerson, "Prudence." Public domain. First published in 1841.
What right have I to write on prudence, of
which I have little, and that of the negative sort?
My prudence consists in avoiding and going
without, not in the inventing of means and
(5) methods, not in adroit steering, not in gentle
repairing. I have no skill to make money spend
well, no genius in my economy, and whoever sees
my garden discovers that I must have some other
garden. Yet I love facts, and hate shiftiness and
(10) people without perception.
Then I have the same title to write on
prudence that I have to write on poetry or
holiness. We write from aspiration as well as from
experience.
(15) We paint those qualities that we do not
possess. The poet admires the man of energy
and tactics; the merchant breeds his son for the
church or the bar; and where a man is not vain and
egotistic you shall find what he lacks, by his praise.
(20) Yet it would be hardly honest for me not
to balance these fine lyric words with words
of coarser sound. Prudence is the virtue of the
senses. It is the science of appearances. It is the
outmost action of the inward life. It is God taking
(25) thought for oxen. It moves matter after the laws
of matter. It is content to seek health of body by
complying with physical conditions, and health
of mind by the laws of the intellect.
The world of the senses is a world of shows;
(30) it does not exist for itself, but has a symbolic
character; and a true prudence or law of shows
recognizes the co-presence of other laws and
knows that its own office is secondary; knows
that it is surface and not center where it works.
(35) Prudence is false when detached. It is legitimate
when it is the natural history of the soul
incarnate, when it unfolds the beauty of laws
within the narrow scope of the senses.
There are all degrees of proficiency in
(40) knowledge of the world. It is sufficient to our
present purpose to indicate three. One class lives
to the utility of the symbol, esteeming health and
wealth a final good. Another class lives above this
mark, to the beauty of the symbol, as the poet and
(45) artist and the naturalist and man of science.
A third class lives above the beauty of the symbol
to the beauty of the thing signified; these are
wise men. The first class has common sense; the
second, taste; and the third, spiritual perception.
(50) Once in a long time, a man traverses the whole
scale, and sees and enjoys the symbol solidly, then
also has a clear eye for its beauty, and lastly, while
he pitches his tent on this sacred volcanic isle of
nature, does not offer to build houses and barns
(55) thereon, reverencing the splendor of the God
which he sees bursting through each chink and
cranny.
The world is filled with the proverbs and
acts of a base prudence, which is a devotion to
(60) matter, as if we possessed no other faculties than
the palate, the nose, the touch, the eye and ear; a
prudence that never subscribes, that never gives,
that seldom lends, and asks but one question of
any project: will it bake bread? This is a disease
(65) like a thickening of the skin until the vital organs
are destroyed. But culture, revealing the high
origin of the apparent world and aiming at the
perfection of the man as the end, degrades every
thing else, as health and bodily life, into means.
(70) It sees prudence not to be a separate faculty,
but a name for wisdom and virtue conversing
with the body and its wants. Cultivated men
always feel and speak so, as if a great fortune, the
achievement of a civil or social measure, great
(75) personal influence, a graceful and commanding
address, had their value as proofs of the energy
of the spirit. If a man loses his balance and
immerses himself in any trades or pleasures for
their own sake, he may be a good wheel or pin,
(80) but he is not a cultivated man.

Q. In line 59, “base” most nearly means

Detailed Solution for OneTime: Digital SAT Mock Test - 5 - Question 8

In saying that the world is filled with the proverbs and acts of a base prudence (lines 58-59), Emerson means that most of our actions and words are devoted to practical things, like the question will it bake bread (lines 64)? As Emerson made clear in his previous paragraph, these considerations are those of the lowest and least noble class, so theirs is an ignoble prudence.

OneTime: Digital SAT Mock Test - 5 - Question 9

Question based on the following passage.
This passage is from Ralph Waldo Emerson, "Prudence." Public domain. First published in 1841.
What right have I to write on prudence, of
which I have little, and that of the negative sort?
My prudence consists in avoiding and going
without, not in the inventing of means and
(5) methods, not in adroit steering, not in gentle
repairing. I have no skill to make money spend
well, no genius in my economy, and whoever sees
my garden discovers that I must have some other
garden. Yet I love facts, and hate shiftiness and
(10) people without perception.
Then I have the same title to write on
prudence that I have to write on poetry or
holiness. We write from aspiration as well as from
experience.
(15) We paint those qualities that we do not
possess. The poet admires the man of energy
and tactics; the merchant breeds his son for the
church or the bar; and where a man is not vain and
egotistic you shall find what he lacks, by his praise.
(20) Yet it would be hardly honest for me not
to balance these fine lyric words with words
of coarser sound. Prudence is the virtue of the
senses. It is the science of appearances. It is the
outmost action of the inward life. It is God taking
(25) thought for oxen. It moves matter after the laws
of matter. It is content to seek health of body by
complying with physical conditions, and health
of mind by the laws of the intellect.
The world of the senses is a world of shows;
(30) it does not exist for itself, but has a symbolic
character; and a true prudence or law of shows
recognizes the co-presence of other laws and
knows that its own office is secondary; knows
that it is surface and not center where it works.
(35) Prudence is false when detached. It is legitimate
when it is the natural history of the soul
incarnate, when it unfolds the beauty of laws
within the narrow scope of the senses.
There are all degrees of proficiency in
(40) knowledge of the world. It is sufficient to our
present purpose to indicate three. One class lives
to the utility of the symbol, esteeming health and
wealth a final good. Another class lives above this
mark, to the beauty of the symbol, as the poet and
(45) artist and the naturalist and man of science.
A third class lives above the beauty of the symbol
to the beauty of the thing signified; these are
wise men. The first class has common sense; the
second, taste; and the third, spiritual perception.
(50) Once in a long time, a man traverses the whole
scale, and sees and enjoys the symbol solidly, then
also has a clear eye for its beauty, and lastly, while
he pitches his tent on this sacred volcanic isle of
nature, does not offer to build houses and barns
(55) thereon, reverencing the splendor of the God
which he sees bursting through each chink and
cranny.
The world is filled with the proverbs and
acts of a base prudence, which is a devotion to
(60) matter, as if we possessed no other faculties than
the palate, the nose, the touch, the eye and ear; a
prudence that never subscribes, that never gives,
that seldom lends, and asks but one question of
any project: will it bake bread? This is a disease
(65) like a thickening of the skin until the vital organs
are destroyed. But culture, revealing the high
origin of the apparent world and aiming at the
perfection of the man as the end, degrades every
thing else, as health and bodily life, into means.
(70) It sees prudence not to be a separate faculty,
but a name for wisdom and virtue conversing
with the body and its wants. Cultivated men
always feel and speak so, as if a great fortune, the
achievement of a civil or social measure, great
(75) personal influence, a graceful and commanding
address, had their value as proofs of the energy
of the spirit. If a man loses his balance and
immerses himself in any trades or pleasures for
their own sake, he may be a good wheel or pin,
(80) but he is not a cultivated man.

Q. The “disease” mentioned in line 64 is best described as

Detailed Solution for OneTime: Digital SAT Mock Test - 5 - Question 9

As a whole, this paragraph discusses the problem that the world is filled with the proverbs and acts of a base prudence (lines 58-59), in other words, that our words and actions are too focused on a devotion to matter (lines 59-60) and its effect on our senses, as if we possessed no other faculties than the palate, the nose, the eye and ear (lines 60-61). Emerson describes this problem with a simile: this is a disease like a thickening of the skin until the vital organs are destroyed (lines 64-66). To Emerson, then, the disease is the problem of sensuousness (devotion to the senses rather than the intellect).

OneTime: Digital SAT Mock Test - 5 - Question 10

Question based on the following passage.
This passage is from Ralph Waldo Emerson, "Prudence." Public domain. First published in 1841.
What right have I to write on prudence, of
which I have little, and that of the negative sort?
My prudence consists in avoiding and going
without, not in the inventing of means and
(5) methods, not in adroit steering, not in gentle
repairing. I have no skill to make money spend
well, no genius in my economy, and whoever sees
my garden discovers that I must have some other
garden. Yet I love facts, and hate shiftiness and
(10) people without perception.
Then I have the same title to write on
prudence that I have to write on poetry or
holiness. We write from aspiration as well as from
experience.
(15) We paint those qualities that we do not
possess. The poet admires the man of energy
and tactics; the merchant breeds his son for the
church or the bar; and where a man is not vain and
egotistic you shall find what he lacks, by his praise.
(20) Yet it would be hardly honest for me not
to balance these fine lyric words with words
of coarser sound. Prudence is the virtue of the
senses. It is the science of appearances. It is the
outmost action of the inward life. It is God taking
(25) thought for oxen. It moves matter after the laws
of matter. It is content to seek health of body by
complying with physical conditions, and health
of mind by the laws of the intellect.
The world of the senses is a world of shows;
(30) it does not exist for itself, but has a symbolic
character; and a true prudence or law of shows
recognizes the co-presence of other laws and
knows that its own office is secondary; knows
that it is surface and not center where it works.
(35) Prudence is false when detached. It is legitimate
when it is the natural history of the soul
incarnate, when it unfolds the beauty of laws
within the narrow scope of the senses.
There are all degrees of proficiency in
(40) knowledge of the world. It is sufficient to our
present purpose to indicate three. One class lives
to the utility of the symbol, esteeming health and
wealth a final good. Another class lives above this
mark, to the beauty of the symbol, as the poet and
(45) artist and the naturalist and man of science.
A third class lives above the beauty of the symbol
to the beauty of the thing signified; these are
wise men. The first class has common sense; the
second, taste; and the third, spiritual perception.
(50) Once in a long time, a man traverses the whole
scale, and sees and enjoys the symbol solidly, then
also has a clear eye for its beauty, and lastly, while
he pitches his tent on this sacred volcanic isle of
nature, does not offer to build houses and barns
(55) thereon, reverencing the splendor of the God
which he sees bursting through each chink and
cranny.
The world is filled with the proverbs and
acts of a base prudence, which is a devotion to
(60) matter, as if we possessed no other faculties than
the palate, the nose, the touch, the eye and ear; a
prudence that never subscribes, that never gives,
that seldom lends, and asks but one question of
any project: will it bake bread? This is a disease
(65) like a thickening of the skin until the vital organs
are destroyed. But culture, revealing the high
origin of the apparent world and aiming at the
perfection of the man as the end, degrades every
thing else, as health and bodily life, into means.
(70) It sees prudence not to be a separate faculty,
but a name for wisdom and virtue conversing
with the body and its wants. Cultivated men
always feel and speak so, as if a great fortune, the
achievement of a civil or social measure, great
(75) personal influence, a graceful and commanding
address, had their value as proofs of the energy
of the spirit. If a man loses his balance and
immerses himself in any trades or pleasures for
their own sake, he may be a good wheel or pin,
(80) but he is not a cultivated man.

Q. The passage as a whole characterizes prudence primarily as.

Detailed Solution for OneTime: Digital SAT Mock Test - 5 - Question 10

In line 20, Emerson defines prudence as the virtue of the senses, but he regards the world of the senses [as] a world of shows (lines 22-23), that is false when detached (line 35) from the thing signified (line 47) by the natural, sensory, intellectual world, that is, from the splendor of God (lines 55). Furthermore, he says that prudence is a devotion to matter, as if we possessed no other faculties than the palate, the nose, the touch, the eye and ear (lines 59-61). Therefore, as a whole, the passage characterizes prudence as a pursuit of practical skills and sensory experience.

OneTime: Digital SAT Mock Test - 5 - Question 11

Question based on the following passage.
This passage is from Joseph Conrad, The Secret Sharer. It was originally published in 1912. The narrator of this story is the captain of a ship about to begin a voyage.
She floated at the starting point of a long
journey, very still in an immense stillness, the
shadows of her spars flung far to the eastward by
the setting sun. At that moment I was alone on her
(5) decks. There was not a sound in her—and around
us nothing moved, nothing lived, not a canoe
on the water, not a bird in the air, not a cloud in
the sky. In this breathless pause at the threshold
of a long passage we seemed to be measuring
(10) our fitness for a long and arduous enterprise,
the appointed task of both our existences to be
carried out, far from all human eyes, with only sky
and sea for spectators and for judges.
There must have been some glare in the
(15) air to interfere with one's sight, because it was
only just before the sun left us that my roaming
eyes made out beyond the highest ridges of the
principal islet of the group something that did
away with the solemnity of perfect solitude.
(20) The tide of darkness flowed on swiftly; and with
tropical suddenness a swarm of stars came out
above the shadowy earth, while I lingered yet,
my hand resting lightly on my ship's rail as if on
the shoulder of a trusted friend. But, with all that
(25) multitude of celestial bodies staring down at one,
the comfort of quiet communion with her was
gone for good. And there were also disturbing
sounds by this time—voices, footsteps forward;
the steward flitted along the main-deck, a busily
(30) ministering spirit; a hand bell tinkled urgently
under the poop deck.
I found my two officers waiting for me near
the supper table, in the lighted cuddy. We sat
down at once, and as I helped the chief mate, I
(35) said: “Are you aware that there is a ship anchored
inside the islands? I saw her mastheads above the
ridge as the sun went down.”
He raised sharply his simple face,
overcharged by a terrible growth of whisker, and
(40) emitted his usual ejaculations:
“Bless my soul, sir! You don't say so!”
My second mate was a round-cheeked, silent
young man, grave beyond his years, I thought;
but as our eyes happened to meet I detected a
(45) slight quiver on his lips. I looked down at once. It
was not my part to encourage sneering on board
my ship. It must be said, too, that I knew very
little of my officers. In consequence of certain
events of no particular significance, except to
(50) myself, I had been appointed to the command
only a fortnight before. Neither did I know much
of the hands forward. All these people had been
together for eighteen months or so, and my
position was that of the only stranger on board.
(55) I mention this because it has some bearing on
what is to follow. But what I felt most was my
being a stranger to the ship; and if all the truth
must be told, I was somewhat of a stranger to
myself. The youngest man on board (barring the
(60) second mate), and untried as yet by a position
of the fullest responsibility, I was willing to take
the adequacy of the others for granted. They had
simply to be equal to their tasks. But I wondered
how far I should turn out faithful to that ideal
(65) conception of one's own personality every man
sets up for himself secretly.
Meantime the chief mate, with an almost
visible effect of collaboration on the part of his
round eyes and frightful whiskers, was trying
(70) to evolve a theory of the anchored ship. His
dominant trait was to take all things into earnest
consideration. He was of a painstaking turn of
mind. As he used to say, he “liked to account to
himself” for practically everything that came
(75) in his way, down to a miserable scorpion he had
found in his cabin a week before. The why and
the wherefore of that scorpion—how it got on
board and came to select his room rather than the
pantry (which was a dark place and more what a
(80) scorpion would be partial to), and how on earth
it managed to drown itself in the inkwell of his
writing desk—had exercised him infinitely.
The ship within the islands was much more
easily accounted for.

Q. The tone of the first paragraph (lines 1–13) is primarily one of

Detailed Solution for OneTime: Digital SAT Mock Test - 5 - Question 11

The opening paragraph describes this breathless pause at the threshold ofa long passage (lines 8-9) in which the narrator and his crew seemed to be measuring our fitness for a long and arduous enterprise (lines 9-10). This describes the reflective anticipation of a journey. Notice that this description provides no evidence of anxiety or excitement. In fact, the scene is described in peaceful terms, with the ship very still in an immense stillness (line 2).

OneTime: Digital SAT Mock Test - 5 - Question 12

Question based on the following passage.
This passage is from Joseph Conrad, The Secret Sharer. It was originally published in 1912. The narrator of this story is the captain of a ship about to begin a voyage.
She floated at the starting point of a long
journey, very still in an immense stillness, the
shadows of her spars flung far to the eastward by
the setting sun. At that moment I was alone on her
(5) decks. There was not a sound in her—and around
us nothing moved, nothing lived, not a canoe
on the water, not a bird in the air, not a cloud in
the sky. In this breathless pause at the threshold
of a long passage we seemed to be measuring
(10) our fitness for a long and arduous enterprise,
the appointed task of both our existences to be
carried out, far from all human eyes, with only sky
and sea for spectators and for judges.
There must have been some glare in the
(15) air to interfere with one's sight, because it was
only just before the sun left us that my roaming
eyes made out beyond the highest ridges of the
principal islet of the group something that did
away with the solemnity of perfect solitude.
(20) The tide of darkness flowed on swiftly; and with
tropical suddenness a swarm of stars came out
above the shadowy earth, while I lingered yet,
my hand resting lightly on my ship's rail as if on
the shoulder of a trusted friend. But, with all that
(25) multitude of celestial bodies staring down at one,
the comfort of quiet communion with her was
gone for good. And there were also disturbing
sounds by this time—voices, footsteps forward;
the steward flitted along the main-deck, a busily
(30) ministering spirit; a hand bell tinkled urgently
under the poop deck.
I found my two officers waiting for me near
the supper table, in the lighted cuddy. We sat
down at once, and as I helped the chief mate, I
(35) said: “Are you aware that there is a ship anchored
inside the islands? I saw her mastheads above the
ridge as the sun went down.”
He raised sharply his simple face,
overcharged by a terrible growth of whisker, and
(40) emitted his usual ejaculations:
“Bless my soul, sir! You don't say so!”
My second mate was a round-cheeked, silent
young man, grave beyond his years, I thought;
but as our eyes happened to meet I detected a
(45) slight quiver on his lips. I looked down at once. It
was not my part to encourage sneering on board
my ship. It must be said, too, that I knew very
little of my officers. In consequence of certain
events of no particular significance, except to
(50) myself, I had been appointed to the command
only a fortnight before. Neither did I know much
of the hands forward. All these people had been
together for eighteen months or so, and my
position was that of the only stranger on board.
(55) I mention this because it has some bearing on
what is to follow. But what I felt most was my
being a stranger to the ship; and if all the truth
must be told, I was somewhat of a stranger to
myself. The youngest man on board (barring the
(60) second mate), and untried as yet by a position
of the fullest responsibility, I was willing to take
the adequacy of the others for granted. They had
simply to be equal to their tasks. But I wondered
how far I should turn out faithful to that ideal
(65) conception of one's own personality every man
sets up for himself secretly.
Meantime the chief mate, with an almost
visible effect of collaboration on the part of his
round eyes and frightful whiskers, was trying
(70) to evolve a theory of the anchored ship. His
dominant trait was to take all things into earnest
consideration. He was of a painstaking turn of
mind. As he used to say, he “liked to account to
himself” for practically everything that came
(75) in his way, down to a miserable scorpion he had
found in his cabin a week before. The why and
the wherefore of that scorpion—how it got on
board and came to select his room rather than the
pantry (which was a dark place and more what a
(80) scorpion would be partial to), and how on earth
it managed to drown itself in the inkwell of his
writing desk—had exercised him infinitely.
The ship within the islands was much more
easily accounted for.

Q. The reference to “some glare” (line 14) serves primarily to make the point that

Detailed Solution for OneTime: Digital SAT Mock Test - 5 - Question 12

The narrator states that some glare in the air (lines 14-15) prevented him from seeing sooner something that did away with the solemnity of perfect solitude (lines 18-19). That is, he saw something that led him to believe they were not alone. In the next paragraph, this something is revealed to be the mastheads of a ship anchored inside the islands (lines 35-36).

OneTime: Digital SAT Mock Test - 5 - Question 13

Question based on the following passage.
This passage is from Joseph Conrad, The Secret Sharer. It was originally published in 1912. The narrator of this story is the captain of a ship about to begin a voyage.
She floated at the starting point of a long
journey, very still in an immense stillness, the
shadows of her spars flung far to the eastward by
the setting sun. At that moment I was alone on her
(5) decks. There was not a sound in her—and around
us nothing moved, nothing lived, not a canoe
on the water, not a bird in the air, not a cloud in
the sky. In this breathless pause at the threshold
of a long passage we seemed to be measuring
(10) our fitness for a long and arduous enterprise,
the appointed task of both our existences to be
carried out, far from all human eyes, with only sky
and sea for spectators and for judges.
There must have been some glare in the
(15) air to interfere with one's sight, because it was
only just before the sun left us that my roaming
eyes made out beyond the highest ridges of the
principal islet of the group something that did
away with the solemnity of perfect solitude.
(20) The tide of darkness flowed on swiftly; and with
tropical suddenness a swarm of stars came out
above the shadowy earth, while I lingered yet,
my hand resting lightly on my ship's rail as if on
the shoulder of a trusted friend. But, with all that
(25) multitude of celestial bodies staring down at one,
the comfort of quiet communion with her was
gone for good. And there were also disturbing
sounds by this time—voices, footsteps forward;
the steward flitted along the main-deck, a busily
(30) ministering spirit; a hand bell tinkled urgently
under the poop deck.
I found my two officers waiting for me near
the supper table, in the lighted cuddy. We sat
down at once, and as I helped the chief mate, I
(35) said: “Are you aware that there is a ship anchored
inside the islands? I saw her mastheads above the
ridge as the sun went down.”
He raised sharply his simple face,
overcharged by a terrible growth of whisker, and
(40) emitted his usual ejaculations:
“Bless my soul, sir! You don't say so!”
My second mate was a round-cheeked, silent
young man, grave beyond his years, I thought;
but as our eyes happened to meet I detected a
(45) slight quiver on his lips. I looked down at once. It
was not my part to encourage sneering on board
my ship. It must be said, too, that I knew very
little of my officers. In consequence of certain
events of no particular significance, except to
(50) myself, I had been appointed to the command
only a fortnight before. Neither did I know much
of the hands forward. All these people had been
together for eighteen months or so, and my
position was that of the only stranger on board.
(55) I mention this because it has some bearing on
what is to follow. But what I felt most was my
being a stranger to the ship; and if all the truth
must be told, I was somewhat of a stranger to
myself. The youngest man on board (barring the
(60) second mate), and untried as yet by a position
of the fullest responsibility, I was willing to take
the adequacy of the others for granted. They had
simply to be equal to their tasks. But I wondered
how far I should turn out faithful to that ideal
(65) conception of one's own personality every man
sets up for himself secretly.
Meantime the chief mate, with an almost
visible effect of collaboration on the part of his
round eyes and frightful whiskers, was trying
(70) to evolve a theory of the anchored ship. His
dominant trait was to take all things into earnest
consideration. He was of a painstaking turn of
mind. As he used to say, he “liked to account to
himself” for practically everything that came
(75) in his way, down to a miserable scorpion he had
found in his cabin a week before. The why and
the wherefore of that scorpion—how it got on
board and came to select his room rather than the
pantry (which was a dark place and more what a
(80) scorpion would be partial to), and how on earth
it managed to drown itself in the inkwell of his
writing desk—had exercised him infinitely.
The ship within the islands was much more
easily accounted for.

Q. In lines 20–24 (“The tide . . . friend”) the narrator describes

Detailed Solution for OneTime: Digital SAT Mock Test - 5 - Question 13

This sentence describes the scene as the narrator surveys the tide of darkness and a swarm of stars (lines 20-21) while resting his hand on the rail of the ship as if it were the shoulder of a trusted friend (line 24). In the next sentence, he describes this as a moment of quiet communion (line 26) with the ship, now interrupted by the sight of a strange ship beyond and the disturbing sounds (lines 27-28) being made by the crew. In other words, this sentence describes a moment of wistful (expressing vague longing) contemplation. Choice (A) is incorrect because, although the disturbing sounds and the omen of a distant ship may seem to be signs of impending danger, the sentence in lines 20-24 makes no mention of these things. Choice (B) is incorrect, because this moment is described as a moment of quiet communion, not deep inner turmoil. Choice (D) is incorrect, because there is no mention of any tragic experience.

OneTime: Digital SAT Mock Test - 5 - Question 14

Question based on the following passage.
This passage is from Joseph Conrad, The Secret Sharer. It was originally published in 1912. The narrator of this story is the captain of a ship about to begin a voyage.
She floated at the starting point of a long
journey, very still in an immense stillness, the
shadows of her spars flung far to the eastward by
the setting sun. At that moment I was alone on her
(5) decks. There was not a sound in her—and around
us nothing moved, nothing lived, not a canoe
on the water, not a bird in the air, not a cloud in
the sky. In this breathless pause at the threshold
of a long passage we seemed to be measuring
(10) our fitness for a long and arduous enterprise,
the appointed task of both our existences to be
carried out, far from all human eyes, with only sky
and sea for spectators and for judges.
There must have been some glare in the
(15) air to interfere with one's sight, because it was
only just before the sun left us that my roaming
eyes made out beyond the highest ridges of the
principal islet of the group something that did
away with the solemnity of perfect solitude.
(20) The tide of darkness flowed on swiftly; and with
tropical suddenness a swarm of stars came out
above the shadowy earth, while I lingered yet,
my hand resting lightly on my ship's rail as if on
the shoulder of a trusted friend. But, with all that
(25) multitude of celestial bodies staring down at one,
the comfort of quiet communion with her was
gone for good. And there were also disturbing
sounds by this time—voices, footsteps forward;
the steward flitted along the main-deck, a busily
(30) ministering spirit; a hand bell tinkled urgently
under the poop deck.
I found my two officers waiting for me near
the supper table, in the lighted cuddy. We sat
down at once, and as I helped the chief mate, I
(35) said: “Are you aware that there is a ship anchored
inside the islands? I saw her mastheads above the
ridge as the sun went down.”
He raised sharply his simple face,
overcharged by a terrible growth of whisker, and
(40) emitted his usual ejaculations:
“Bless my soul, sir! You don't say so!”
My second mate was a round-cheeked, silent
young man, grave beyond his years, I thought;
but as our eyes happened to meet I detected a
(45) slight quiver on his lips. I looked down at once. It
was not my part to encourage sneering on board
my ship. It must be said, too, that I knew very
little of my officers. In consequence of certain
events of no particular significance, except to
(50) myself, I had been appointed to the command
only a fortnight before. Neither did I know much
of the hands forward. All these people had been
together for eighteen months or so, and my
position was that of the only stranger on board.
(55) I mention this because it has some bearing on
what is to follow. But what I felt most was my
being a stranger to the ship; and if all the truth
must be told, I was somewhat of a stranger to
myself. The youngest man on board (barring the
(60) second mate), and untried as yet by a position
of the fullest responsibility, I was willing to take
the adequacy of the others for granted. They had
simply to be equal to their tasks. But I wondered
how far I should turn out faithful to that ideal
(65) conception of one's own personality every man
sets up for himself secretly.
Meantime the chief mate, with an almost
visible effect of collaboration on the part of his
round eyes and frightful whiskers, was trying
(70) to evolve a theory of the anchored ship. His
dominant trait was to take all things into earnest
consideration. He was of a painstaking turn of
mind. As he used to say, he “liked to account to
himself” for practically everything that came
(75) in his way, down to a miserable scorpion he had
found in his cabin a week before. The why and
the wherefore of that scorpion—how it got on
board and came to select his room rather than the
pantry (which was a dark place and more what a
(80) scorpion would be partial to), and how on earth
it managed to drown itself in the inkwell of his
writing desk—had exercised him infinitely.
The ship within the islands was much more
easily accounted for.

Q. The captain is portrayed primarily as

Detailed Solution for OneTime: Digital SAT Mock Test - 5 - Question 14

Since this story is being told from the perspective of the captain, we can infer his character from the nature of his narration. In the opening paragraph, the captain states that we seemed to be measuring our fitness for a long arduous enterprise, the point of our existences to be carried out (lines 9-12), demonstrating that he is more reflective than reactive as a leader. Much later he says, what I felt most was my being a stranger to the ship; and if all the truth must be told, I was somewhat of a stranger to myself... I wondered how far I should turn out faithful to that ideal conception of one's own personality every man sets up for himself secretly (lines 56-66). These descriptions of reflection and self-doubt reveal the captain as being self-conscious and diffident.

OneTime: Digital SAT Mock Test - 5 - Question 15

Question based on the following passage.
This passage is from Joseph Conrad, The Secret Sharer. It was originally published in 1912. The narrator of this story is the captain of a ship about to begin a voyage.
She floated at the starting point of a long
journey, very still in an immense stillness, the
shadows of her spars flung far to the eastward by
the setting sun. At that moment I was alone on her
(5) decks. There was not a sound in her—and around
us nothing moved, nothing lived, not a canoe
on the water, not a bird in the air, not a cloud in
the sky. In this breathless pause at the threshold
of a long passage we seemed to be measuring
(10) our fitness for a long and arduous enterprise,
the appointed task of both our existences to be
carried out, far from all human eyes, with only sky
and sea for spectators and for judges.
There must have been some glare in the
(15) air to interfere with one's sight, because it was
only just before the sun left us that my roaming
eyes made out beyond the highest ridges of the
principal islet of the group something that did
away with the solemnity of perfect solitude.
(20) The tide of darkness flowed on swiftly; and with
tropical suddenness a swarm of stars came out
above the shadowy earth, while I lingered yet,
my hand resting lightly on my ship's rail as if on
the shoulder of a trusted friend. But, with all that
(25) multitude of celestial bodies staring down at one,
the comfort of quiet communion with her was
gone for good. And there were also disturbing
sounds by this time—voices, footsteps forward;
the steward flitted along the main-deck, a busily
(30) ministering spirit; a hand bell tinkled urgently
under the poop deck.
I found my two officers waiting for me near
the supper table, in the lighted cuddy. We sat
down at once, and as I helped the chief mate, I
(35) said: “Are you aware that there is a ship anchored
inside the islands? I saw her mastheads above the
ridge as the sun went down.”
He raised sharply his simple face,
overcharged by a terrible growth of whisker, and
(40) emitted his usual ejaculations:
“Bless my soul, sir! You don't say so!”
My second mate was a round-cheeked, silent
young man, grave beyond his years, I thought;
but as our eyes happened to meet I detected a
(45) slight quiver on his lips. I looked down at once. It
was not my part to encourage sneering on board
my ship. It must be said, too, that I knew very
little of my officers. In consequence of certain
events of no particular significance, except to
(50) myself, I had been appointed to the command
only a fortnight before. Neither did I know much
of the hands forward. All these people had been
together for eighteen months or so, and my
position was that of the only stranger on board.
(55) I mention this because it has some bearing on
what is to follow. But what I felt most was my
being a stranger to the ship; and if all the truth
must be told, I was somewhat of a stranger to
myself. The youngest man on board (barring the
(60) second mate), and untried as yet by a position
of the fullest responsibility, I was willing to take
the adequacy of the others for granted. They had
simply to be equal to their tasks. But I wondered
how far I should turn out faithful to that ideal
(65) conception of one's own personality every man
sets up for himself secretly.
Meantime the chief mate, with an almost
visible effect of collaboration on the part of his
round eyes and frightful whiskers, was trying
(70) to evolve a theory of the anchored ship. His
dominant trait was to take all things into earnest
consideration. He was of a painstaking turn of
mind. As he used to say, he “liked to account to
himself” for practically everything that came
(75) in his way, down to a miserable scorpion he had
found in his cabin a week before. The why and
the wherefore of that scorpion—how it got on
board and came to select his room rather than the
pantry (which was a dark place and more what a
(80) scorpion would be partial to), and how on earth
it managed to drown itself in the inkwell of his
writing desk—had exercised him infinitely.
The ship within the islands was much more
easily accounted for.

Q. Which choice provides the strongest evidence for the answer to the previous question?

Detailed Solution for OneTime: Digital SAT Mock Test - 5 - Question 15

As the explanation to question 14 shows, the best evidence for this answer can be found in lines 63-65.

OneTime: Digital SAT Mock Test - 5 - Question 16

Question based on the following passage.
This passage is from Joseph Conrad, The Secret Sharer. It was originally published in 1912. The narrator of this story is the captain of a ship about to begin a voyage.
She floated at the starting point of a long
journey, very still in an immense stillness, the
shadows of her spars flung far to the eastward by
the setting sun. At that moment I was alone on her
(5) decks. There was not a sound in her—and around
us nothing moved, nothing lived, not a canoe
on the water, not a bird in the air, not a cloud in
the sky. In this breathless pause at the threshold
of a long passage we seemed to be measuring
(10) our fitness for a long and arduous enterprise,
the appointed task of both our existences to be
carried out, far from all human eyes, with only sky
and sea for spectators and for judges.
There must have been some glare in the
(15) air to interfere with one's sight, because it was
only just before the sun left us that my roaming
eyes made out beyond the highest ridges of the
principal islet of the group something that did
away with the solemnity of perfect solitude.
(20) The tide of darkness flowed on swiftly; and with
tropical suddenness a swarm of stars came out
above the shadowy earth, while I lingered yet,
my hand resting lightly on my ship's rail as if on
the shoulder of a trusted friend. But, with all that
(25) multitude of celestial bodies staring down at one,
the comfort of quiet communion with her was
gone for good. And there were also disturbing
sounds by this time—voices, footsteps forward;
the steward flitted along the main-deck, a busily
(30) ministering spirit; a hand bell tinkled urgently
under the poop deck.
I found my two officers waiting for me near
the supper table, in the lighted cuddy. We sat
down at once, and as I helped the chief mate, I
(35) said: “Are you aware that there is a ship anchored
inside the islands? I saw her mastheads above the
ridge as the sun went down.”
He raised sharply his simple face,
overcharged by a terrible growth of whisker, and
(40) emitted his usual ejaculations:
“Bless my soul, sir! You don't say so!”
My second mate was a round-cheeked, silent
young man, grave beyond his years, I thought;
but as our eyes happened to meet I detected a
(45) slight quiver on his lips. I looked down at once. It
was not my part to encourage sneering on board
my ship. It must be said, too, that I knew very
little of my officers. In consequence of certain
events of no particular significance, except to
(50) myself, I had been appointed to the command
only a fortnight before. Neither did I know much
of the hands forward. All these people had been
together for eighteen months or so, and my
position was that of the only stranger on board.
(55) I mention this because it has some bearing on
what is to follow. But what I felt most was my
being a stranger to the ship; and if all the truth
must be told, I was somewhat of a stranger to
myself. The youngest man on board (barring the
(60) second mate), and untried as yet by a position
of the fullest responsibility, I was willing to take
the adequacy of the others for granted. They had
simply to be equal to their tasks. But I wondered
how far I should turn out faithful to that ideal
(65) conception of one's own personality every man
sets up for himself secretly.
Meantime the chief mate, with an almost
visible effect of collaboration on the part of his
round eyes and frightful whiskers, was trying
(70) to evolve a theory of the anchored ship. His
dominant trait was to take all things into earnest
consideration. He was of a painstaking turn of
mind. As he used to say, he “liked to account to
himself” for practically everything that came
(75) in his way, down to a miserable scorpion he had
found in his cabin a week before. The why and
the wherefore of that scorpion—how it got on
board and came to select his room rather than the
pantry (which was a dark place and more what a
(80) scorpion would be partial to), and how on earth
it managed to drown itself in the inkwell of his
writing desk—had exercised him infinitely.
The ship within the islands was much more
easily accounted for.

Q. In line 55, “bearing” most nearly means

Detailed Solution for OneTime: Digital SAT Mock Test - 5 - Question 16

In saying I mention this because it has some bearing on what is to follow (lines 55-56), the narrator means that the fact that he was the only stranger on board (line 54) is relevant to what he is about to say.

OneTime: Digital SAT Mock Test - 5 - Question 17

Question based on the following passage.
This passage is from Joseph Conrad, The Secret Sharer. It was originally published in 1912. The narrator of this story is the captain of a ship about to begin a voyage.
She floated at the starting point of a long
journey, very still in an immense stillness, the
shadows of her spars flung far to the eastward by
the setting sun. At that moment I was alone on her
(5) decks. There was not a sound in her—and around
us nothing moved, nothing lived, not a canoe
on the water, not a bird in the air, not a cloud in
the sky. In this breathless pause at the threshold
of a long passage we seemed to be measuring
(10) our fitness for a long and arduous enterprise,
the appointed task of both our existences to be
carried out, far from all human eyes, with only sky
and sea for spectators and for judges.
There must have been some glare in the
(15) air to interfere with one's sight, because it was
only just before the sun left us that my roaming
eyes made out beyond the highest ridges of the
principal islet of the group something that did
away with the solemnity of perfect solitude.
(20) The tide of darkness flowed on swiftly; and with
tropical suddenness a swarm of stars came out
above the shadowy earth, while I lingered yet,
my hand resting lightly on my ship's rail as if on
the shoulder of a trusted friend. But, with all that
(25) multitude of celestial bodies staring down at one,
the comfort of quiet communion with her was
gone for good. And there were also disturbing
sounds by this time—voices, footsteps forward;
the steward flitted along the main-deck, a busily
(30) ministering spirit; a hand bell tinkled urgently
under the poop deck.
I found my two officers waiting for me near
the supper table, in the lighted cuddy. We sat
down at once, and as I helped the chief mate, I
(35) said: “Are you aware that there is a ship anchored
inside the islands? I saw her mastheads above the
ridge as the sun went down.”
He raised sharply his simple face,
overcharged by a terrible growth of whisker, and
(40) emitted his usual ejaculations:
“Bless my soul, sir! You don't say so!”
My second mate was a round-cheeked, silent
young man, grave beyond his years, I thought;
but as our eyes happened to meet I detected a
(45) slight quiver on his lips. I looked down at once. It
was not my part to encourage sneering on board
my ship. It must be said, too, that I knew very
little of my officers. In consequence of certain
events of no particular significance, except to
(50) myself, I had been appointed to the command
only a fortnight before. Neither did I know much
of the hands forward. All these people had been
together for eighteen months or so, and my
position was that of the only stranger on board.
(55) I mention this because it has some bearing on
what is to follow. But what I felt most was my
being a stranger to the ship; and if all the truth
must be told, I was somewhat of a stranger to
myself. The youngest man on board (barring the
(60) second mate), and untried as yet by a position
of the fullest responsibility, I was willing to take
the adequacy of the others for granted. They had
simply to be equal to their tasks. But I wondered
how far I should turn out faithful to that ideal
(65) conception of one's own personality every man
sets up for himself secretly.
Meantime the chief mate, with an almost
visible effect of collaboration on the part of his
round eyes and frightful whiskers, was trying
(70) to evolve a theory of the anchored ship. His
dominant trait was to take all things into earnest
consideration. He was of a painstaking turn of
mind. As he used to say, he “liked to account to
himself” for practically everything that came
(75) in his way, down to a miserable scorpion he had
found in his cabin a week before. The why and
the wherefore of that scorpion—how it got on
board and came to select his room rather than the
pantry (which was a dark place and more what a
(80) scorpion would be partial to), and how on earth
it managed to drown itself in the inkwell of his
writing desk—had exercised him infinitely.
The ship within the islands was much more
easily accounted for.

Q. In line 70, “evolve” most nearly means

Detailed Solution for OneTime: Digital SAT Mock Test - 5 - Question 17

This sentence describes how the chief mate, described as earnest (line 71) and painstaking (72), is trying strenuously to figure out why there is another ship anchored nearby. In saying that he was trying to evolve a theory, the narrator means he is pondering (thinking) strenuously.

OneTime: Digital SAT Mock Test - 5 - Question 18

Question based on the following passage.
This passage is from Joseph Conrad, The Secret Sharer. It was originally published in 1912. The narrator of this story is the captain of a ship about to begin a voyage.
She floated at the starting point of a long
journey, very still in an immense stillness, the
shadows of her spars flung far to the eastward by
the setting sun. At that moment I was alone on her
(5) decks. There was not a sound in her—and around
us nothing moved, nothing lived, not a canoe
on the water, not a bird in the air, not a cloud in
the sky. In this breathless pause at the threshold
of a long passage we seemed to be measuring
(10) our fitness for a long and arduous enterprise,
the appointed task of both our existences to be
carried out, far from all human eyes, with only sky
and sea for spectators and for judges.
There must have been some glare in the
(15) air to interfere with one's sight, because it was
only just before the sun left us that my roaming
eyes made out beyond the highest ridges of the
principal islet of the group something that did
away with the solemnity of perfect solitude.
(20) The tide of darkness flowed on swiftly; and with
tropical suddenness a swarm of stars came out
above the shadowy earth, while I lingered yet,
my hand resting lightly on my ship's rail as if on
the shoulder of a trusted friend. But, with all that
(25) multitude of celestial bodies staring down at one,
the comfort of quiet communion with her was
gone for good. And there were also disturbing
sounds by this time—voices, footsteps forward;
the steward flitted along the main-deck, a busily
(30) ministering spirit; a hand bell tinkled urgently
under the poop deck.
I found my two officers waiting for me near
the supper table, in the lighted cuddy. We sat
down at once, and as I helped the chief mate, I
(35) said: “Are you aware that there is a ship anchored
inside the islands? I saw her mastheads above the
ridge as the sun went down.”
He raised sharply his simple face,
overcharged by a terrible growth of whisker, and
(40) emitted his usual ejaculations:
“Bless my soul, sir! You don't say so!”
My second mate was a round-cheeked, silent
young man, grave beyond his years, I thought;
but as our eyes happened to meet I detected a
(45) slight quiver on his lips. I looked down at once. It
was not my part to encourage sneering on board
my ship. It must be said, too, that I knew very
little of my officers. In consequence of certain
events of no particular significance, except to
(50) myself, I had been appointed to the command
only a fortnight before. Neither did I know much
of the hands forward. All these people had been
together for eighteen months or so, and my
position was that of the only stranger on board.
(55) I mention this because it has some bearing on
what is to follow. But what I felt most was my
being a stranger to the ship; and if all the truth
must be told, I was somewhat of a stranger to
myself. The youngest man on board (barring the
(60) second mate), and untried as yet by a position
of the fullest responsibility, I was willing to take
the adequacy of the others for granted. They had
simply to be equal to their tasks. But I wondered
how far I should turn out faithful to that ideal
(65) conception of one's own personality every man
sets up for himself secretly.
Meantime the chief mate, with an almost
visible effect of collaboration on the part of his
round eyes and frightful whiskers, was trying
(70) to evolve a theory of the anchored ship. His
dominant trait was to take all things into earnest
consideration. He was of a painstaking turn of
mind. As he used to say, he “liked to account to
himself” for practically everything that came
(75) in his way, down to a miserable scorpion he had
found in his cabin a week before. The why and
the wherefore of that scorpion—how it got on
board and came to select his room rather than the
pantry (which was a dark place and more what a
(80) scorpion would be partial to), and how on earth
it managed to drown itself in the inkwell of his
writing desk—had exercised him infinitely.
The ship within the islands was much more
easily accounted for.

Q. The “truth” to which the narrator refers in lines 57 is his

Detailed Solution for OneTime: Digital SAT Mock Test - 5 - Question 18

The truth that the narrator mentions in line 57 is the fact that I am a stranger to myself. He later goes on to explain what he means by this: I wondered how far I should turn out faithful to that ideal conception of one's own personality every man sets up for himself secretly (lines 63-66). In other words, this truth is the fact that he lacks self-confidence.

OneTime: Digital SAT Mock Test - 5 - Question 19

Question based on the following passage.
This passage is from Joseph Conrad, The Secret Sharer. It was originally published in 1912. The narrator of this story is the captain of a ship about to begin a voyage.
She floated at the starting point of a long
journey, very still in an immense stillness, the
shadows of her spars flung far to the eastward by
the setting sun. At that moment I was alone on her
(5) decks. There was not a sound in her—and around
us nothing moved, nothing lived, not a canoe
on the water, not a bird in the air, not a cloud in
the sky. In this breathless pause at the threshold
of a long passage we seemed to be measuring
(10) our fitness for a long and arduous enterprise,
the appointed task of both our existences to be
carried out, far from all human eyes, with only sky
and sea for spectators and for judges.
There must have been some glare in the
(15) air to interfere with one's sight, because it was
only just before the sun left us that my roaming
eyes made out beyond the highest ridges of the
principal islet of the group something that did
away with the solemnity of perfect solitude.
(20) The tide of darkness flowed on swiftly; and with
tropical suddenness a swarm of stars came out
above the shadowy earth, while I lingered yet,
my hand resting lightly on my ship's rail as if on
the shoulder of a trusted friend. But, with all that
(25) multitude of celestial bodies staring down at one,
the comfort of quiet communion with her was
gone for good. And there were also disturbing
sounds by this time—voices, footsteps forward;
the steward flitted along the main-deck, a busily
(30) ministering spirit; a hand bell tinkled urgently
under the poop deck.
I found my two officers waiting for me near
the supper table, in the lighted cuddy. We sat
down at once, and as I helped the chief mate, I
(35) said: “Are you aware that there is a ship anchored
inside the islands? I saw her mastheads above the
ridge as the sun went down.”
He raised sharply his simple face,
overcharged by a terrible growth of whisker, and
(40) emitted his usual ejaculations:
“Bless my soul, sir! You don't say so!”
My second mate was a round-cheeked, silent
young man, grave beyond his years, I thought;
but as our eyes happened to meet I detected a
(45) slight quiver on his lips. I looked down at once. It
was not my part to encourage sneering on board
my ship. It must be said, too, that I knew very
little of my officers. In consequence of certain
events of no particular significance, except to
(50) myself, I had been appointed to the command
only a fortnight before. Neither did I know much
of the hands forward. All these people had been
together for eighteen months or so, and my
position was that of the only stranger on board.
(55) I mention this because it has some bearing on
what is to follow. But what I felt most was my
being a stranger to the ship; and if all the truth
must be told, I was somewhat of a stranger to
myself. The youngest man on board (barring the
(60) second mate), and untried as yet by a position
of the fullest responsibility, I was willing to take
the adequacy of the others for granted. They had
simply to be equal to their tasks. But I wondered
how far I should turn out faithful to that ideal
(65) conception of one's own personality every man
sets up for himself secretly.
Meantime the chief mate, with an almost
visible effect of collaboration on the part of his
round eyes and frightful whiskers, was trying
(70) to evolve a theory of the anchored ship. His
dominant trait was to take all things into earnest
consideration. He was of a painstaking turn of
mind. As he used to say, he “liked to account to
himself” for practically everything that came
(75) in his way, down to a miserable scorpion he had
found in his cabin a week before. The why and
the wherefore of that scorpion—how it got on
board and came to select his room rather than the
pantry (which was a dark place and more what a
(80) scorpion would be partial to), and how on earth
it managed to drown itself in the inkwell of his
writing desk—had exercised him infinitely.
The ship within the islands was much more
easily accounted for.

Q. In line 82, “exercised” most nearly means

Detailed Solution for OneTime: Digital SAT Mock Test - 5 - Question 19

In saying that the why and the wherefore of that scorpion . . . had exercised him infinitely (lines 76-82), the narrator means that the chief mate was using his dominant trait . . . [of] earnest consideration (lines 71-72) to figure out how a scorpion had made its way into his cabin. That is, the questions about the scorpion had disquieted (unsettled) him infinitely.

OneTime: Digital SAT Mock Test - 5 - Question 20

Question based on the following passage.
This passage is from Joseph Conrad, The Secret Sharer. It was originally published in 1912. The narrator of this story is the captain of a ship about to begin a voyage.
She floated at the starting point of a long
journey, very still in an immense stillness, the
shadows of her spars flung far to the eastward by
the setting sun. At that moment I was alone on her
(5) decks. There was not a sound in her—and around
us nothing moved, nothing lived, not a canoe
on the water, not a bird in the air, not a cloud in
the sky. In this breathless pause at the threshold
of a long passage we seemed to be measuring
(10) our fitness for a long and arduous enterprise,
the appointed task of both our existences to be
carried out, far from all human eyes, with only sky
and sea for spectators and for judges.
There must have been some glare in the
(15) air to interfere with one's sight, because it was
only just before the sun left us that my roaming
eyes made out beyond the highest ridges of the
principal islet of the group something that did
away with the solemnity of perfect solitude.
(20) The tide of darkness flowed on swiftly; and with
tropical suddenness a swarm of stars came out
above the shadowy earth, while I lingered yet,
my hand resting lightly on my ship's rail as if on
the shoulder of a trusted friend. But, with all that
(25) multitude of celestial bodies staring down at one,
the comfort of quiet communion with her was
gone for good. And there were also disturbing
sounds by this time—voices, footsteps forward;
the steward flitted along the main-deck, a busily
(30) ministering spirit; a hand bell tinkled urgently
under the poop deck.
I found my two officers waiting for me near
the supper table, in the lighted cuddy. We sat
down at once, and as I helped the chief mate, I
(35) said: “Are you aware that there is a ship anchored
inside the islands? I saw her mastheads above the
ridge as the sun went down.”
He raised sharply his simple face,
overcharged by a terrible growth of whisker, and
(40) emitted his usual ejaculations:
“Bless my soul, sir! You don't say so!”
My second mate was a round-cheeked, silent
young man, grave beyond his years, I thought;
but as our eyes happened to meet I detected a
(45) slight quiver on his lips. I looked down at once. It
was not my part to encourage sneering on board
my ship. It must be said, too, that I knew very
little of my officers. In consequence of certain
events of no particular significance, except to
(50) myself, I had been appointed to the command
only a fortnight before. Neither did I know much
of the hands forward. All these people had been
together for eighteen months or so, and my
position was that of the only stranger on board.
(55) I mention this because it has some bearing on
what is to follow. But what I felt most was my
being a stranger to the ship; and if all the truth
must be told, I was somewhat of a stranger to
myself. The youngest man on board (barring the
(60) second mate), and untried as yet by a position
of the fullest responsibility, I was willing to take
the adequacy of the others for granted. They had
simply to be equal to their tasks. But I wondered
how far I should turn out faithful to that ideal
(65) conception of one's own personality every man
sets up for himself secretly.
Meantime the chief mate, with an almost
visible effect of collaboration on the part of his
round eyes and frightful whiskers, was trying
(70) to evolve a theory of the anchored ship. His
dominant trait was to take all things into earnest
consideration. He was of a painstaking turn of
mind. As he used to say, he “liked to account to
himself” for practically everything that came
(75) in his way, down to a miserable scorpion he had
found in his cabin a week before. The why and
the wherefore of that scorpion—how it got on
board and came to select his room rather than the
pantry (which was a dark place and more what a
(80) scorpion would be partial to), and how on earth
it managed to drown itself in the inkwell of his
writing desk—had exercised him infinitely.
The ship within the islands was much more
easily accounted for.

Q. The “collaboration” (line 68) refers to an act of

Detailed Solution for OneTime: Digital SAT Mock Test - 5 - Question 20

The collaboration on the part of [the chief mate's] round eyes and frightful whiskers (lines 67-68) describes his facial contortions as he deliberates about the anchored ship. In other words, it is an act of strained contemplation.

OneTime: Digital SAT Mock Test - 5 - Question 21

Question based on the following passage.
This passage is from Joseph Conrad, The Secret Sharer. It was originally published in 1912. The narrator of this story is the captain of a ship about to begin a voyage.
She floated at the starting point of a long
journey, very still in an immense stillness, the
shadows of her spars flung far to the eastward by
the setting sun. At that moment I was alone on her
(5) decks. There was not a sound in her—and around
us nothing moved, nothing lived, not a canoe
on the water, not a bird in the air, not a cloud in
the sky. In this breathless pause at the threshold
of a long passage we seemed to be measuring
(10) our fitness for a long and arduous enterprise,
the appointed task of both our existences to be
carried out, far from all human eyes, with only sky
and sea for spectators and for judges.
There must have been some glare in the
(15) air to interfere with one's sight, because it was
only just before the sun left us that my roaming
eyes made out beyond the highest ridges of the
principal islet of the group something that did
away with the solemnity of perfect solitude.
(20) The tide of darkness flowed on swiftly; and with
tropical suddenness a swarm of stars came out
above the shadowy earth, while I lingered yet,
my hand resting lightly on my ship's rail as if on
the shoulder of a trusted friend. But, with all that
(25) multitude of celestial bodies staring down at one,
the comfort of quiet communion with her was
gone for good. And there were also disturbing
sounds by this time—voices, footsteps forward;
the steward flitted along the main-deck, a busily
(30) ministering spirit; a hand bell tinkled urgently
under the poop deck.
I found my two officers waiting for me near
the supper table, in the lighted cuddy. We sat
down at once, and as I helped the chief mate, I
(35) said: “Are you aware that there is a ship anchored
inside the islands? I saw her mastheads above the
ridge as the sun went down.”
He raised sharply his simple face,
overcharged by a terrible growth of whisker, and
(40) emitted his usual ejaculations:
“Bless my soul, sir! You don't say so!”
My second mate was a round-cheeked, silent
young man, grave beyond his years, I thought;
but as our eyes happened to meet I detected a
(45) slight quiver on his lips. I looked down at once. It
was not my part to encourage sneering on board
my ship. It must be said, too, that I knew very
little of my officers. In consequence of certain
events of no particular significance, except to
(50) myself, I had been appointed to the command
only a fortnight before. Neither did I know much
of the hands forward. All these people had been
together for eighteen months or so, and my
position was that of the only stranger on board.
(55) I mention this because it has some bearing on
what is to follow. But what I felt most was my
being a stranger to the ship; and if all the truth
must be told, I was somewhat of a stranger to
myself. The youngest man on board (barring the
(60) second mate), and untried as yet by a position
of the fullest responsibility, I was willing to take
the adequacy of the others for granted. They had
simply to be equal to their tasks. But I wondered
how far I should turn out faithful to that ideal
(65) conception of one's own personality every man
sets up for himself secretly.
Meantime the chief mate, with an almost
visible effect of collaboration on the part of his
round eyes and frightful whiskers, was trying
(70) to evolve a theory of the anchored ship. His
dominant trait was to take all things into earnest
consideration. He was of a painstaking turn of
mind. As he used to say, he “liked to account to
himself” for practically everything that came
(75) in his way, down to a miserable scorpion he had
found in his cabin a week before. The why and
the wherefore of that scorpion—how it got on
board and came to select his room rather than the
pantry (which was a dark place and more what a
(80) scorpion would be partial to), and how on earth
it managed to drown itself in the inkwell of his
writing desk—had exercised him infinitely.
The ship within the islands was much more
easily accounted for.

Q. The chief mate believed that, compared to the recently discovered ship, the “scorpion” (line 75) was

Detailed Solution for OneTime: Digital SAT Mock Test - 5 - Question 21

In the final line, the narrator says that the ship within the islands was much more easily accounted for. In other words, the scorpion was less easily accounted for, or less explicable.

OneTime: Digital SAT Mock Test - 5 - Question 22

Question based on the following passages.
Passage 1 is from Lindsay Smith-Doyle, "Thoughts on the Value of Life." ©2015 by College Hill Coaching. Passage 2 is from C. F. Black, “Who's Afraid of Cloning?" ©2015 by College Hill Coaching. Since 1996, when scientists at the Roslin Institute in England cloned a sheep from the cells of another adult sheep, many have debated the ethics of cloning human cells. These passages are excerpts from arguments on this issue.
Passage 1
How should human life be bestowed? With
human cloning looming as a real scientific
possibility, we must question the provenance of
this ultimate gift. Our intimate participation in
(5) the creation of life must never be misconstrued as
control. Rather, our attitude toward the creation
of life must be one of humility.
The idea of “outsourcing” the creation of
human life, of relegating it to a laboratory, of
(10) reducing the anticipation of childbirth to a trip
to the mall or a selection from a catalog, mocks
the profundity of life. The mystery is replaced by
design and control. Should we turn our noses up
at the most precious gift in the universe, only to
(15) say: “Sorry, but I think I can do better?”
Cloning is the engineering of human life. We
have for the first time the ability to determine the
exact genetic makeup of a human being. Whether
you believe in evolution or creationism, cloning
(20) thwarts an essential step of the conception
process: randomness in the case of natural
selection, and guided purpose in the case of
creationism. A child can be created that is no
longer uniquely human but the end product of an
(25) assembly line, with carefully designed and tested
features. Are the astonishing processes of nature
somehow deficient?
If human cloning becomes acceptable, we will
have created a new society in which the value of
(30) human life is marginalized. Industries will arise
that turn human procreation into a profitable
free-market enterprise. The executive boards of
these companies will decide the course of human
evolution, with more concern for quarterly profit
(35) reports than for the fate of humanity.
These are not idle concerns. Even as we
ponder the ethical implications of human
cloning, companies are forging ahead with
procedures to clone human cells for seemingly
(40) beneficial purposes, marching steadily toward
a Brave New World in which humanity will be
forever less human.
Passage 2
The breathless fears about human cloning
should not surprise anyone who knows the
(45) history of science. Every step in human progress
is met with close-mindedness that often verges on
paranoia. Not even medicine is spared.
As doctors toil to save, prolong, and improve
lives, the uninformed rage at the arrogance
(50) of science. Before the merits of surgery and
vaccination became commonplace and obvious,
many refused to believe that cutting flesh or
introducing degraded germs could do more good
than harm. Perhaps we should turn from science
(55) and return to superstition and magic spells?
At first glance, it might seem that cloning is
a whole new ballgame. After all, cloning is “the
engineering of human life,” isn't it? It is the mass
production of designer babies. It is the end of
(60) evolution, or at least the beginning of its corporate
management. It is certainly a slap in the face of
God. Or is it?
Cloning foe Jeremy Rifkin is afraid of nothing
so much as duplication: “It's a horrendous crime
(65) to make a Xerox of someone. You're putting a
human into a genetic straitjacket.” The horror! I
wonder how Mr. Rifkin would feel at the annual
Twins Days Festival in Twinsburg, Ohio. Genetic
Xeroxes everywhere!
(70) Identical twins are not monsters. Rifkin's
fear is vacuous. Each identical twin has his or
her own unique thoughts, talents, experiences,
and beliefs. Mr. Rifkin must learn that human
beings are more than just their DNA; they are
(75) the products of the continual and inscrutably
complex interactions of environment and biology.
Human clones would be no different.
“But you are playing God!” we hear. It is the
cry of all whose power is threatened by the march
(80) of human progress. It is the reasoning of the Dark
Ages, used to keep the subservient masses in their
place. Every great step humanity has ever taken
has disrupted the "natural order." Should we be
shivering in caves, eating uncooked bugs, and
(85) dying of parasites, as nature intended?
But perhaps procreation is different—more
sacred. Then why have the technologies of fertility
enhancement, in vitro fertilization, embryo
transfer, and birth control become so widely
(90) accepted? Each of these technologies was met at
first with legions of strident opponents. But over
time, reality and compassion overcame unreason
and paranoia. Familiarity dissipates fear.
These supposedly "moral" objections are
(95) in fact impeding moral progress. With genetic
engineering, cloning, and stem cell research,
scientists finally have within their grasp
technologies that can provide ample food for
a starving world, cure devastating illnesses,
(100) and replace diseased organs. Only ignorant
superstition stands in their way.

Q. In line 13, “control” refers specifically to control over

Detailed Solution for OneTime: Digital SAT Mock Test - 5 - Question 22

The second paragraph discusses the "outsourcing” [of] the creation of human life (lines 8-9), so the design and control mentioned in line 13 refer specifically to the design and control of the process of conception.

OneTime: Digital SAT Mock Test - 5 - Question 23

Question based on the following passages.
Passage 1 is from Lindsay Smith-Doyle, "Thoughts on the Value of Life." ©2015 by College Hill Coaching. Passage 2 is from C. F. Black, “Who's Afraid of Cloning?" ©2015 by College Hill Coaching. Since 1996, when scientists at the Roslin Institute in England cloned a sheep from the cells of another adult sheep, many have debated the ethics of cloning human cells. These passages are excerpts from arguments on this issue.
Passage 1
How should human life be bestowed? With
human cloning looming as a real scientific
possibility, we must question the provenance of
this ultimate gift. Our intimate participation in
(5) the creation of life must never be misconstrued as
control. Rather, our attitude toward the creation
of life must be one of humility.
The idea of “outsourcing” the creation of
human life, of relegating it to a laboratory, of
(10) reducing the anticipation of childbirth to a trip
to the mall or a selection from a catalog, mocks
the profundity of life. The mystery is replaced by
design and control. Should we turn our noses up
at the most precious gift in the universe, only to
(15) say: “Sorry, but I think I can do better?”
Cloning is the engineering of human life. We
have for the first time the ability to determine the
exact genetic makeup of a human being. Whether
you believe in evolution or creationism, cloning
(20) thwarts an essential step of the conception
process: randomness in the case of natural
selection, and guided purpose in the case of
creationism. A child can be created that is no
longer uniquely human but the end product of an
(25) assembly line, with carefully designed and tested
features. Are the astonishing processes of nature
somehow deficient?
If human cloning becomes acceptable, we will
have created a new society in which the value of
(30) human life is marginalized. Industries will arise
that turn human procreation into a profitable
free-market enterprise. The executive boards of
these companies will decide the course of human
evolution, with more concern for quarterly profit
(35) reports than for the fate of humanity.
These are not idle concerns. Even as we
ponder the ethical implications of human
cloning, companies are forging ahead with
procedures to clone human cells for seemingly
(40) beneficial purposes, marching steadily toward
a Brave New World in which humanity will be
forever less human.
Passage 2
The breathless fears about human cloning
should not surprise anyone who knows the
(45) history of science. Every step in human progress
is met with close-mindedness that often verges on
paranoia. Not even medicine is spared.
As doctors toil to save, prolong, and improve
lives, the uninformed rage at the arrogance
(50) of science. Before the merits of surgery and
vaccination became commonplace and obvious,
many refused to believe that cutting flesh or
introducing degraded germs could do more good
than harm. Perhaps we should turn from science
(55) and return to superstition and magic spells?
At first glance, it might seem that cloning is
a whole new ballgame. After all, cloning is “the
engineering of human life,” isn't it? It is the mass
production of designer babies. It is the end of
(60) evolution, or at least the beginning of its corporate
management. It is certainly a slap in the face of
God. Or is it?
Cloning foe Jeremy Rifkin is afraid of nothing
so much as duplication: “It's a horrendous crime
(65) to make a Xerox of someone. You're putting a
human into a genetic straitjacket.” The horror! I
wonder how Mr. Rifkin would feel at the annual
Twins Days Festival in Twinsburg, Ohio. Genetic
Xeroxes everywhere!
(70) Identical twins are not monsters. Rifkin's
fear is vacuous. Each identical twin has his or
her own unique thoughts, talents, experiences,
and beliefs. Mr. Rifkin must learn that human
beings are more than just their DNA; they are
(75) the products of the continual and inscrutably
complex interactions of environment and biology.
Human clones would be no different.
“But you are playing God!” we hear. It is the
cry of all whose power is threatened by the march
(80) of human progress. It is the reasoning of the Dark
Ages, used to keep the subservient masses in their
place. Every great step humanity has ever taken
has disrupted the "natural order." Should we be
shivering in caves, eating uncooked bugs, and
(85) dying of parasites, as nature intended?
But perhaps procreation is different—more
sacred. Then why have the technologies of fertility
enhancement, in vitro fertilization, embryo
transfer, and birth control become so widely
(90) accepted? Each of these technologies was met at
first with legions of strident opponents. But over
time, reality and compassion overcame unreason
and paranoia. Familiarity dissipates fear.
These supposedly "moral" objections are
(95) in fact impeding moral progress. With genetic
engineering, cloning, and stem cell research,
scientists finally have within their grasp
technologies that can provide ample food for
a starving world, cure devastating illnesses,
(100) and replace diseased organs. Only ignorant
superstition stands in their way.

Q. In Passage 1, the author’s attitude toward “outsourcing” (line 8) is one of

Detailed Solution for OneTime: Digital SAT Mock Test - 5 - Question 23

The author of Passage 1 states that the "outsourcing” [of] the creation of human life... mocks the profundity of life (lines 8-12) and he provides no indication in the passage that he otherwise approves of it. Clearly, then, he regards it with blunt disdain.

OneTime: Digital SAT Mock Test - 5 - Question 24

Question based on the following passages.
Passage 1 is from Lindsay Smith-Doyle, "Thoughts on the Value of Life." ©2015 by College Hill Coaching. Passage 2 is from C. F. Black, “Who's Afraid of Cloning?" ©2015 by College Hill Coaching. Since 1996, when scientists at the Roslin Institute in England cloned a sheep from the cells of another adult sheep, many have debated the ethics of cloning human cells. These passages are excerpts from arguments on this issue.
Passage 1
How should human life be bestowed? With
human cloning looming as a real scientific
possibility, we must question the provenance of
this ultimate gift. Our intimate participation in
(5) the creation of life must never be misconstrued as
control. Rather, our attitude toward the creation
of life must be one of humility.
The idea of “outsourcing” the creation of
human life, of relegating it to a laboratory, of
(10) reducing the anticipation of childbirth to a trip
to the mall or a selection from a catalog, mocks
the profundity of life. The mystery is replaced by
design and control. Should we turn our noses up
at the most precious gift in the universe, only to
(15) say: “Sorry, but I think I can do better?”
Cloning is the engineering of human life. We
have for the first time the ability to determine the
exact genetic makeup of a human being. Whether
you believe in evolution or creationism, cloning
(20) thwarts an essential step of the conception
process: randomness in the case of natural
selection, and guided purpose in the case of
creationism. A child can be created that is no
longer uniquely human but the end product of an
(25) assembly line, with carefully designed and tested
features. Are the astonishing processes of nature
somehow deficient?
If human cloning becomes acceptable, we will
have created a new society in which the value of
(30) human life is marginalized. Industries will arise
that turn human procreation into a profitable
free-market enterprise. The executive boards of
these companies will decide the course of human
evolution, with more concern for quarterly profit
(35) reports than for the fate of humanity.
These are not idle concerns. Even as we
ponder the ethical implications of human
cloning, companies are forging ahead with
procedures to clone human cells for seemingly
(40) beneficial purposes, marching steadily toward
a Brave New World in which humanity will be
forever less human.
Passage 2
The breathless fears about human cloning
should not surprise anyone who knows the
(45) history of science. Every step in human progress
is met with close-mindedness that often verges on
paranoia. Not even medicine is spared.
As doctors toil to save, prolong, and improve
lives, the uninformed rage at the arrogance
(50) of science. Before the merits of surgery and
vaccination became commonplace and obvious,
many refused to believe that cutting flesh or
introducing degraded germs could do more good
than harm. Perhaps we should turn from science
(55) and return to superstition and magic spells?
At first glance, it might seem that cloning is
a whole new ballgame. After all, cloning is “the
engineering of human life,” isn't it? It is the mass
production of designer babies. It is the end of
(60) evolution, or at least the beginning of its corporate
management. It is certainly a slap in the face of
God. Or is it?
Cloning foe Jeremy Rifkin is afraid of nothing
so much as duplication: “It's a horrendous crime
(65) to make a Xerox of someone. You're putting a
human into a genetic straitjacket.” The horror! I
wonder how Mr. Rifkin would feel at the annual
Twins Days Festival in Twinsburg, Ohio. Genetic
Xeroxes everywhere!
(70) Identical twins are not monsters. Rifkin's
fear is vacuous. Each identical twin has his or
her own unique thoughts, talents, experiences,
and beliefs. Mr. Rifkin must learn that human
beings are more than just their DNA; they are
(75) the products of the continual and inscrutably
complex interactions of environment and biology.
Human clones would be no different.
“But you are playing God!” we hear. It is the
cry of all whose power is threatened by the march
(80) of human progress. It is the reasoning of the Dark
Ages, used to keep the subservient masses in their
place. Every great step humanity has ever taken
has disrupted the "natural order." Should we be
shivering in caves, eating uncooked bugs, and
(85) dying of parasites, as nature intended?
But perhaps procreation is different—more
sacred. Then why have the technologies of fertility
enhancement, in vitro fertilization, embryo
transfer, and birth control become so widely
(90) accepted? Each of these technologies was met at
first with legions of strident opponents. But over
time, reality and compassion overcame unreason
and paranoia. Familiarity dissipates fear.
These supposedly "moral" objections are
(95) in fact impeding moral progress. With genetic
engineering, cloning, and stem cell research,
scientists finally have within their grasp
technologies that can provide ample food for
a starving world, cure devastating illnesses,
(100) and replace diseased organs. Only ignorant
superstition stands in their way.

Q. The quotations in line 15 and line 78 are similar in that both

Detailed Solution for OneTime: Digital SAT Mock Test - 5 - Question 24

Both of these quotations represent viewpoints with which the authors of the respective passages disagree. In Passage 1, the quotation "Sorry, but I think I can do better" (line 15) is from those who turn [their] noses up at the most precious gift in the universe (lines 13-14) much to the chagrin of the author. In Passage 2, the quotation "But you are playing God" (line 78) is described as the cry of all whose power is threatened by the march of human progress, and with whom the author clearly disagrees.

OneTime: Digital SAT Mock Test - 5 - Question 25

Question based on the following passages.
Passage 1 is from Lindsay Smith-Doyle, "Thoughts on the Value of Life." ©2015 by College Hill Coaching. Passage 2 is from C. F. Black, “Who's Afraid of Cloning?" ©2015 by College Hill Coaching. Since 1996, when scientists at the Roslin Institute in England cloned a sheep from the cells of another adult sheep, many have debated the ethics of cloning human cells. These passages are excerpts from arguments on this issue.
Passage 1
How should human life be bestowed? With
human cloning looming as a real scientific
possibility, we must question the provenance of
this ultimate gift. Our intimate participation in
(5) the creation of life must never be misconstrued as
control. Rather, our attitude toward the creation
of life must be one of humility.
The idea of “outsourcing” the creation of
human life, of relegating it to a laboratory, of
(10) reducing the anticipation of childbirth to a trip
to the mall or a selection from a catalog, mocks
the profundity of life. The mystery is replaced by
design and control. Should we turn our noses up
at the most precious gift in the universe, only to
(15) say: “Sorry, but I think I can do better?”
Cloning is the engineering of human life. We
have for the first time the ability to determine the
exact genetic makeup of a human being. Whether
you believe in evolution or creationism, cloning
(20) thwarts an essential step of the conception
process: randomness in the case of natural
selection, and guided purpose in the case of
creationism. A child can be created that is no
longer uniquely human but the end product of an
(25) assembly line, with carefully designed and tested
features. Are the astonishing processes of nature
somehow deficient?
If human cloning becomes acceptable, we will
have created a new society in which the value of
(30) human life is marginalized. Industries will arise
that turn human procreation into a profitable
free-market enterprise. The executive boards of
these companies will decide the course of human
evolution, with more concern for quarterly profit
(35) reports than for the fate of humanity.
These are not idle concerns. Even as we
ponder the ethical implications of human
cloning, companies are forging ahead with
procedures to clone human cells for seemingly
(40) beneficial purposes, marching steadily toward
a Brave New World in which humanity will be
forever less human.
Passage 2
The breathless fears about human cloning
should not surprise anyone who knows the
(45) history of science. Every step in human progress
is met with close-mindedness that often verges on
paranoia. Not even medicine is spared.
As doctors toil to save, prolong, and improve
lives, the uninformed rage at the arrogance
(50) of science. Before the merits of surgery and
vaccination became commonplace and obvious,
many refused to believe that cutting flesh or
introducing degraded germs could do more good
than harm. Perhaps we should turn from science
(55) and return to superstition and magic spells?
At first glance, it might seem that cloning is
a whole new ballgame. After all, cloning is “the
engineering of human life,” isn't it? It is the mass
production of designer babies. It is the end of
(60) evolution, or at least the beginning of its corporate
management. It is certainly a slap in the face of
God. Or is it?
Cloning foe Jeremy Rifkin is afraid of nothing
so much as duplication: “It's a horrendous crime
(65) to make a Xerox of someone. You're putting a
human into a genetic straitjacket.” The horror! I
wonder how Mr. Rifkin would feel at the annual
Twins Days Festival in Twinsburg, Ohio. Genetic
Xeroxes everywhere!
(70) Identical twins are not monsters. Rifkin's
fear is vacuous. Each identical twin has his or
her own unique thoughts, talents, experiences,
and beliefs. Mr. Rifkin must learn that human
beings are more than just their DNA; they are
(75) the products of the continual and inscrutably
complex interactions of environment and biology.
Human clones would be no different.
“But you are playing God!” we hear. It is the
cry of all whose power is threatened by the march
(80) of human progress. It is the reasoning of the Dark
Ages, used to keep the subservient masses in their
place. Every great step humanity has ever taken
has disrupted the "natural order." Should we be
shivering in caves, eating uncooked bugs, and
(85) dying of parasites, as nature intended?
But perhaps procreation is different—more
sacred. Then why have the technologies of fertility
enhancement, in vitro fertilization, embryo
transfer, and birth control become so widely
(90) accepted? Each of these technologies was met at
first with legions of strident opponents. But over
time, reality and compassion overcame unreason
and paranoia. Familiarity dissipates fear.
These supposedly "moral" objections are
(95) in fact impeding moral progress. With genetic
engineering, cloning, and stem cell research,
scientists finally have within their grasp
technologies that can provide ample food for
a starving world, cure devastating illnesses,
(100) and replace diseased organs. Only ignorant
superstition stands in their way.

Q. Jeremy Rifkin (line 63) would most likely advocate

Detailed Solution for OneTime: Digital SAT Mock Test - 5 - Question 25

Jeremy Rifkin is described in Passage 2 as a cloning foe (line 63) who is quoted as saying "It's a horrendous crime to make a Xerox of someone. You're putting a human into a genetic straitjacket. " Presumably, then, he would agree that our attitude toward the creation of life must be one of humility (lines 6-7).

OneTime: Digital SAT Mock Test - 5 - Question 26

Question based on the following passages.
Passage 1 is from Lindsay Smith-Doyle, "Thoughts on the Value of Life." ©2015 by College Hill Coaching. Passage 2 is from C. F. Black, “Who's Afraid of Cloning?" ©2015 by College Hill Coaching. Since 1996, when scientists at the Roslin Institute in England cloned a sheep from the cells of another adult sheep, many have debated the ethics of cloning human cells. These passages are excerpts from arguments on this issue.
Passage 1
How should human life be bestowed? With
human cloning looming as a real scientific
possibility, we must question the provenance of
this ultimate gift. Our intimate participation in
(5) the creation of life must never be misconstrued as
control. Rather, our attitude toward the creation
of life must be one of humility.
The idea of “outsourcing” the creation of
human life, of relegating it to a laboratory, of
(10) reducing the anticipation of childbirth to a trip
to the mall or a selection from a catalog, mocks
the profundity of life. The mystery is replaced by
design and control. Should we turn our noses up
at the most precious gift in the universe, only to
(15) say: “Sorry, but I think I can do better?”
Cloning is the engineering of human life. We
have for the first time the ability to determine the
exact genetic makeup of a human being. Whether
you believe in evolution or creationism, cloning
(20) thwarts an essential step of the conception
process: randomness in the case of natural
selection, and guided purpose in the case of
creationism. A child can be created that is no
longer uniquely human but the end product of an
(25) assembly line, with carefully designed and tested
features. Are the astonishing processes of nature
somehow deficient?
If human cloning becomes acceptable, we will
have created a new society in which the value of
(30) human life is marginalized. Industries will arise
that turn human procreation into a profitable
free-market enterprise. The executive boards of
these companies will decide the course of human
evolution, with more concern for quarterly profit
(35) reports than for the fate of humanity.
These are not idle concerns. Even as we
ponder the ethical implications of human
cloning, companies are forging ahead with
procedures to clone human cells for seemingly
(40) beneficial purposes, marching steadily toward
a Brave New World in which humanity will be
forever less human.
Passage 2
The breathless fears about human cloning
should not surprise anyone who knows the
(45) history of science. Every step in human progress
is met with close-mindedness that often verges on
paranoia. Not even medicine is spared.
As doctors toil to save, prolong, and improve
lives, the uninformed rage at the arrogance
(50) of science. Before the merits of surgery and
vaccination became commonplace and obvious,
many refused to believe that cutting flesh or
introducing degraded germs could do more good
than harm. Perhaps we should turn from science
(55) and return to superstition and magic spells?
At first glance, it might seem that cloning is
a whole new ballgame. After all, cloning is “the
engineering of human life,” isn't it? It is the mass
production of designer babies. It is the end of
(60) evolution, or at least the beginning of its corporate
management. It is certainly a slap in the face of
God. Or is it?
Cloning foe Jeremy Rifkin is afraid of nothing
so much as duplication: “It's a horrendous crime
(65) to make a Xerox of someone. You're putting a
human into a genetic straitjacket.” The horror! I
wonder how Mr. Rifkin would feel at the annual
Twins Days Festival in Twinsburg, Ohio. Genetic
Xeroxes everywhere!
(70) Identical twins are not monsters. Rifkin's
fear is vacuous. Each identical twin has his or
her own unique thoughts, talents, experiences,
and beliefs. Mr. Rifkin must learn that human
beings are more than just their DNA; they are
(75) the products of the continual and inscrutably
complex interactions of environment and biology.
Human clones would be no different.
“But you are playing God!” we hear. It is the
cry of all whose power is threatened by the march
(80) of human progress. It is the reasoning of the Dark
Ages, used to keep the subservient masses in their
place. Every great step humanity has ever taken
has disrupted the "natural order." Should we be
shivering in caves, eating uncooked bugs, and
(85) dying of parasites, as nature intended?
But perhaps procreation is different—more
sacred. Then why have the technologies of fertility
enhancement, in vitro fertilization, embryo
transfer, and birth control become so widely
(90) accepted? Each of these technologies was met at
first with legions of strident opponents. But over
time, reality and compassion overcame unreason
and paranoia. Familiarity dissipates fear.
These supposedly "moral" objections are
(95) in fact impeding moral progress. With genetic
engineering, cloning, and stem cell research,
scientists finally have within their grasp
technologies that can provide ample food for
a starving world, cure devastating illnesses,
(100) and replace diseased organs. Only ignorant
superstition stands in their way.

Q. The diagram best illustrates

Detailed Solution for OneTime: Digital SAT Mock Test - 5 - Question 26

The illustration shows a schematic overview of some Therapeutic Cloning Strategies that involve removing a somatic cell from a patient and transferring its nucleus to stem cells that can then be cultured into genetically matched tissue that can then replace diseased cells and tissues in the patient. This is an example of one of the procedures to clone human cells for seemingly beneficial purposes (lines 39-40) described in Passage 1. Choice (A) is incorrect because the guided purpose refers to a principle of creationism, which is not indicated at all in the diagram. Choice (B) is incorrect because, although the process in the diagram might resemble an assembly line, it is not the assembly line that could be used to create a child that is no longer uniquely human (lines 23-24), but with carefully designed and tested features (lines 25-26). Choice (C) is incorrect because the diagram does not describe the course of human evolution, which would need to show how humans evolved from more primitive species.

OneTime: Digital SAT Mock Test - 5 - Question 27

Question based on the following passages.
Passage 1 is from Lindsay Smith-Doyle, "Thoughts on the Value of Life." ©2015 by College Hill Coaching. Passage 2 is from C. F. Black, “Who's Afraid of Cloning?" ©2015 by College Hill Coaching. Since 1996, when scientists at the Roslin Institute in England cloned a sheep from the cells of another adult sheep, many have debated the ethics of cloning human cells. These passages are excerpts from arguments on this issue.
Passage 1
How should human life be bestowed? With
human cloning looming as a real scientific
possibility, we must question the provenance of
this ultimate gift. Our intimate participation in
(5) the creation of life must never be misconstrued as
control. Rather, our attitude toward the creation
of life must be one of humility.
The idea of “outsourcing” the creation of
human life, of relegating it to a laboratory, of
(10) reducing the anticipation of childbirth to a trip
to the mall or a selection from a catalog, mocks
the profundity of life. The mystery is replaced by
design and control. Should we turn our noses up
at the most precious gift in the universe, only to
(15) say: “Sorry, but I think I can do better?”
Cloning is the engineering of human life. We
have for the first time the ability to determine the
exact genetic makeup of a human being. Whether
you believe in evolution or creationism, cloning
(20) thwarts an essential step of the conception
process: randomness in the case of natural
selection, and guided purpose in the case of
creationism. A child can be created that is no
longer uniquely human but the end product of an
(25) assembly line, with carefully designed and tested
features. Are the astonishing processes of nature
somehow deficient?
If human cloning becomes acceptable, we will
have created a new society in which the value of
(30) human life is marginalized. Industries will arise
that turn human procreation into a profitable
free-market enterprise. The executive boards of
these companies will decide the course of human
evolution, with more concern for quarterly profit
(35) reports than for the fate of humanity.
These are not idle concerns. Even as we
ponder the ethical implications of human
cloning, companies are forging ahead with
procedures to clone human cells for seemingly
(40) beneficial purposes, marching steadily toward
a Brave New World in which humanity will be
forever less human.
Passage 2
The breathless fears about human cloning
should not surprise anyone who knows the
(45) history of science. Every step in human progress
is met with close-mindedness that often verges on
paranoia. Not even medicine is spared.
As doctors toil to save, prolong, and improve
lives, the uninformed rage at the arrogance
(50) of science. Before the merits of surgery and
vaccination became commonplace and obvious,
many refused to believe that cutting flesh or
introducing degraded germs could do more good
than harm. Perhaps we should turn from science
(55) and return to superstition and magic spells?
At first glance, it might seem that cloning is
a whole new ballgame. After all, cloning is “the
engineering of human life,” isn't it? It is the mass
production of designer babies. It is the end of
(60) evolution, or at least the beginning of its corporate
management. It is certainly a slap in the face of
God. Or is it?
Cloning foe Jeremy Rifkin is afraid of nothing
so much as duplication: “It's a horrendous crime
(65) to make a Xerox of someone. You're putting a
human into a genetic straitjacket.” The horror! I
wonder how Mr. Rifkin would feel at the annual
Twins Days Festival in Twinsburg, Ohio. Genetic
Xeroxes everywhere!
(70) Identical twins are not monsters. Rifkin's
fear is vacuous. Each identical twin has his or
her own unique thoughts, talents, experiences,
and beliefs. Mr. Rifkin must learn that human
beings are more than just their DNA; they are
(75) the products of the continual and inscrutably
complex interactions of environment and biology.
Human clones would be no different.
“But you are playing God!” we hear. It is the
cry of all whose power is threatened by the march
(80) of human progress. It is the reasoning of the Dark
Ages, used to keep the subservient masses in their
place. Every great step humanity has ever taken
has disrupted the "natural order." Should we be
shivering in caves, eating uncooked bugs, and
(85) dying of parasites, as nature intended?
But perhaps procreation is different—more
sacred. Then why have the technologies of fertility
enhancement, in vitro fertilization, embryo
transfer, and birth control become so widely
(90) accepted? Each of these technologies was met at
first with legions of strident opponents. But over
time, reality and compassion overcame unreason
and paranoia. Familiarity dissipates fear.
These supposedly "moral" objections are
(95) in fact impeding moral progress. With genetic
engineering, cloning, and stem cell research,
scientists finally have within their grasp
technologies that can provide ample food for
a starving world, cure devastating illnesses,
(100) and replace diseased organs. Only ignorant
superstition stands in their way.

Q. In line 53, “introducing” refers to an act of

Detailed Solution for OneTime: Digital SAT Mock Test - 5 - Question 27

The process of introducing degraded germs (line 53) describes the basic process of vaccination, which, like cutting flesh (line 52) (that is, surgery), must have seemed dangerous at first, but in fact can be a life-saving technology. This process is the injection of vaccines.

OneTime: Digital SAT Mock Test - 5 - Question 28

Question based on the following passages.
Passage 1 is from Lindsay Smith-Doyle, "Thoughts on the Value of Life." ©2015 by College Hill Coaching. Passage 2 is from C. F. Black, “Who's Afraid of Cloning?" ©2015 by College Hill Coaching. Since 1996, when scientists at the Roslin Institute in England cloned a sheep from the cells of another adult sheep, many have debated the ethics of cloning human cells. These passages are excerpts from arguments on this issue.
Passage 1
How should human life be bestowed? With
human cloning looming as a real scientific
possibility, we must question the provenance of
this ultimate gift. Our intimate participation in
(5) the creation of life must never be misconstrued as
control. Rather, our attitude toward the creation
of life must be one of humility.
The idea of “outsourcing” the creation of
human life, of relegating it to a laboratory, of
(10) reducing the anticipation of childbirth to a trip
to the mall or a selection from a catalog, mocks
the profundity of life. The mystery is replaced by
design and control. Should we turn our noses up
at the most precious gift in the universe, only to
(15) say: “Sorry, but I think I can do better?”
Cloning is the engineering of human life. We
have for the first time the ability to determine the
exact genetic makeup of a human being. Whether
you believe in evolution or creationism, cloning
(20) thwarts an essential step of the conception
process: randomness in the case of natural
selection, and guided purpose in the case of
creationism. A child can be created that is no
longer uniquely human but the end product of an
(25) assembly line, with carefully designed and tested
features. Are the astonishing processes of nature
somehow deficient?
If human cloning becomes acceptable, we will
have created a new society in which the value of
(30) human life is marginalized. Industries will arise
that turn human procreation into a profitable
free-market enterprise. The executive boards of
these companies will decide the course of human
evolution, with more concern for quarterly profit
(35) reports than for the fate of humanity.
These are not idle concerns. Even as we
ponder the ethical implications of human
cloning, companies are forging ahead with
procedures to clone human cells for seemingly
(40) beneficial purposes, marching steadily toward
a Brave New World in which humanity will be
forever less human.
Passage 2
The breathless fears about human cloning
should not surprise anyone who knows the
(45) history of science. Every step in human progress
is met with close-mindedness that often verges on
paranoia. Not even medicine is spared.
As doctors toil to save, prolong, and improve
lives, the uninformed rage at the arrogance
(50) of science. Before the merits of surgery and
vaccination became commonplace and obvious,
many refused to believe that cutting flesh or
introducing degraded germs could do more good
than harm. Perhaps we should turn from science
(55) and return to superstition and magic spells?
At first glance, it might seem that cloning is
a whole new ballgame. After all, cloning is “the
engineering of human life,” isn't it? It is the mass
production of designer babies. It is the end of
(60) evolution, or at least the beginning of its corporate
management. It is certainly a slap in the face of
God. Or is it?
Cloning foe Jeremy Rifkin is afraid of nothing
so much as duplication: “It's a horrendous crime
(65) to make a Xerox of someone. You're putting a
human into a genetic straitjacket.” The horror! I
wonder how Mr. Rifkin would feel at the annual
Twins Days Festival in Twinsburg, Ohio. Genetic
Xeroxes everywhere!
(70) Identical twins are not monsters. Rifkin's
fear is vacuous. Each identical twin has his or
her own unique thoughts, talents, experiences,
and beliefs. Mr. Rifkin must learn that human
beings are more than just their DNA; they are
(75) the products of the continual and inscrutably
complex interactions of environment and biology.
Human clones would be no different.
“But you are playing God!” we hear. It is the
cry of all whose power is threatened by the march
(80) of human progress. It is the reasoning of the Dark
Ages, used to keep the subservient masses in their
place. Every great step humanity has ever taken
has disrupted the "natural order." Should we be
shivering in caves, eating uncooked bugs, and
(85) dying of parasites, as nature intended?
But perhaps procreation is different—more
sacred. Then why have the technologies of fertility
enhancement, in vitro fertilization, embryo
transfer, and birth control become so widely
(90) accepted? Each of these technologies was met at
first with legions of strident opponents. But over
time, reality and compassion overcame unreason
and paranoia. Familiarity dissipates fear.
These supposedly "moral" objections are
(95) in fact impeding moral progress. With genetic
engineering, cloning, and stem cell research,
scientists finally have within their grasp
technologies that can provide ample food for
a starving world, cure devastating illnesses,
(100) and replace diseased organs. Only ignorant
superstition stands in their way.

Q. The author of Passage 1 would most likely regard the “management” (line 61) described in Passage 2 as

Detailed Solution for OneTime: Digital SAT Mock Test - 5 - Question 28

In this paragraph, the author of Passage 2 describes the position of cloning foes who believe that cloning is the end of evolution, or at least the beginning of its corporate management (lines 59-61). The author of Passage 1 is deeply concerned that the executive boards of these [cloning] companies will decide the course of human evolution, with more concern for quarterly profit reports than for the sake of humanity (lines 32-35). Clearly, then, the author of Passage 1 regards this management as a regrettable invasion of commercial interests into human reproduction.

OneTime: Digital SAT Mock Test - 5 - Question 29

Question based on the following passages.
Passage 1 is from Lindsay Smith-Doyle, "Thoughts on the Value of Life." ©2015 by College Hill Coaching. Passage 2 is from C. F. Black, “Who's Afraid of Cloning?" ©2015 by College Hill Coaching. Since 1996, when scientists at the Roslin Institute in England cloned a sheep from the cells of another adult sheep, many have debated the ethics of cloning human cells. These passages are excerpts from arguments on this issue.
Passage 1
How should human life be bestowed? With
human cloning looming as a real scientific
possibility, we must question the provenance of
this ultimate gift. Our intimate participation in
(5) the creation of life must never be misconstrued as
control. Rather, our attitude toward the creation
of life must be one of humility.
The idea of “outsourcing” the creation of
human life, of relegating it to a laboratory, of
(10) reducing the anticipation of childbirth to a trip
to the mall or a selection from a catalog, mocks
the profundity of life. The mystery is replaced by
design and control. Should we turn our noses up
at the most precious gift in the universe, only to
(15) say: “Sorry, but I think I can do better?”
Cloning is the engineering of human life. We
have for the first time the ability to determine the
exact genetic makeup of a human being. Whether
you believe in evolution or creationism, cloning
(20) thwarts an essential step of the conception
process: randomness in the case of natural
selection, and guided purpose in the case of
creationism. A child can be created that is no
longer uniquely human but the end product of an
(25) assembly line, with carefully designed and tested
features. Are the astonishing processes of nature
somehow deficient?
If human cloning becomes acceptable, we will
have created a new society in which the value of
(30) human life is marginalized. Industries will arise
that turn human procreation into a profitable
free-market enterprise. The executive boards of
these companies will decide the course of human
evolution, with more concern for quarterly profit
(35) reports than for the fate of humanity.
These are not idle concerns. Even as we
ponder the ethical implications of human
cloning, companies are forging ahead with
procedures to clone human cells for seemingly
(40) beneficial purposes, marching steadily toward
a Brave New World in which humanity will be
forever less human.
Passage 2
The breathless fears about human cloning
should not surprise anyone who knows the
(45) history of science. Every step in human progress
is met with close-mindedness that often verges on
paranoia. Not even medicine is spared.
As doctors toil to save, prolong, and improve
lives, the uninformed rage at the arrogance
(50) of science. Before the merits of surgery and
vaccination became commonplace and obvious,
many refused to believe that cutting flesh or
introducing degraded germs could do more good
than harm. Perhaps we should turn from science
(55) and return to superstition and magic spells?
At first glance, it might seem that cloning is
a whole new ballgame. After all, cloning is “the
engineering of human life,” isn't it? It is the mass
production of designer babies. It is the end of
(60) evolution, or at least the beginning of its corporate
management. It is certainly a slap in the face of
God. Or is it?
Cloning foe Jeremy Rifkin is afraid of nothing
so much as duplication: “It's a horrendous crime
(65) to make a Xerox of someone. You're putting a
human into a genetic straitjacket.” The horror! I
wonder how Mr. Rifkin would feel at the annual
Twins Days Festival in Twinsburg, Ohio. Genetic
Xeroxes everywhere!
(70) Identical twins are not monsters. Rifkin's
fear is vacuous. Each identical twin has his or
her own unique thoughts, talents, experiences,
and beliefs. Mr. Rifkin must learn that human
beings are more than just their DNA; they are
(75) the products of the continual and inscrutably
complex interactions of environment and biology.
Human clones would be no different.
“But you are playing God!” we hear. It is the
cry of all whose power is threatened by the march
(80) of human progress. It is the reasoning of the Dark
Ages, used to keep the subservient masses in their
place. Every great step humanity has ever taken
has disrupted the "natural order." Should we be
shivering in caves, eating uncooked bugs, and
(85) dying of parasites, as nature intended?
But perhaps procreation is different—more
sacred. Then why have the technologies of fertility
enhancement, in vitro fertilization, embryo
transfer, and birth control become so widely
(90) accepted? Each of these technologies was met at
first with legions of strident opponents. But over
time, reality and compassion overcame unreason
and paranoia. Familiarity dissipates fear.
These supposedly "moral" objections are
(95) in fact impeding moral progress. With genetic
engineering, cloning, and stem cell research,
scientists finally have within their grasp
technologies that can provide ample food for
a starving world, cure devastating illnesses,
(100) and replace diseased organs. Only ignorant
superstition stands in their way.

Q. Passage 2 quotes Jeremy Rifkin in lines 64–66 primarily to

Detailed Solution for OneTime: Digital SAT Mock Test - 5 - Question 29

Jeremy Rifkin's belief that cloning is a horrendous crime (line 64) directly contradicts the thesis of Passage 2, which is that cloning and similar technologies can provide ample food for a starving world, cure devastating illnesses, and replace diseased organs (lines 98-100). Therefore, to the author of Passage 2, Rifkin's opinion exemplifies an untenable (indefensible) position. Choice (B) may seem plausible, since Rifkin is warning of the potential dangers of cloning, but notice that this cannot be the reason that the author of Passage 2 quotes Rifkin, because the passage clearly disagrees with his sentiments.

OneTime: Digital SAT Mock Test - 5 - Question 30

Question based on the following passages.
Passage 1 is from Lindsay Smith-Doyle, "Thoughts on the Value of Life." ©2015 by College Hill Coaching. Passage 2 is from C. F. Black, “Who's Afraid of Cloning?" ©2015 by College Hill Coaching. Since 1996, when scientists at the Roslin Institute in England cloned a sheep from the cells of another adult sheep, many have debated the ethics of cloning human cells. These passages are excerpts from arguments on this issue.
Passage 1
How should human life be bestowed? With
human cloning looming as a real scientific
possibility, we must question the provenance of
this ultimate gift. Our intimate participation in
(5) the creation of life must never be misconstrued as
control. Rather, our attitude toward the creation
of life must be one of humility.
The idea of “outsourcing” the creation of
human life, of relegating it to a laboratory, of
(10) reducing the anticipation of childbirth to a trip
to the mall or a selection from a catalog, mocks
the profundity of life. The mystery is replaced by
design and control. Should we turn our noses up
at the most precious gift in the universe, only to
(15) say: “Sorry, but I think I can do better?”
Cloning is the engineering of human life. We
have for the first time the ability to determine the
exact genetic makeup of a human being. Whether
you believe in evolution or creationism, cloning
(20) thwarts an essential step of the conception
process: randomness in the case of natural
selection, and guided purpose in the case of
creationism. A child can be created that is no
longer uniquely human but the end product of an
(25) assembly line, with carefully designed and tested
features. Are the astonishing processes of nature
somehow deficient?
If human cloning becomes acceptable, we will
have created a new society in which the value of
(30) human life is marginalized. Industries will arise
that turn human procreation into a profitable
free-market enterprise. The executive boards of
these companies will decide the course of human
evolution, with more concern for quarterly profit
(35) reports than for the fate of humanity.
These are not idle concerns. Even as we
ponder the ethical implications of human
cloning, companies are forging ahead with
procedures to clone human cells for seemingly
(40) beneficial purposes, marching steadily toward
a Brave New World in which humanity will be
forever less human.
Passage 2
The breathless fears about human cloning
should not surprise anyone who knows the
(45) history of science. Every step in human progress
is met with close-mindedness that often verges on
paranoia. Not even medicine is spared.
As doctors toil to save, prolong, and improve
lives, the uninformed rage at the arrogance
(50) of science. Before the merits of surgery and
vaccination became commonplace and obvious,
many refused to believe that cutting flesh or
introducing degraded germs could do more good
than harm. Perhaps we should turn from science
(55) and return to superstition and magic spells?
At first glance, it might seem that cloning is
a whole new ballgame. After all, cloning is “the
engineering of human life,” isn't it? It is the mass
production of designer babies. It is the end of
(60) evolution, or at least the beginning of its corporate
management. It is certainly a slap in the face of
God. Or is it?
Cloning foe Jeremy Rifkin is afraid of nothing
so much as duplication: “It's a horrendous crime
(65) to make a Xerox of someone. You're putting a
human into a genetic straitjacket.” The horror! I
wonder how Mr. Rifkin would feel at the annual
Twins Days Festival in Twinsburg, Ohio. Genetic
Xeroxes everywhere!
(70) Identical twins are not monsters. Rifkin's
fear is vacuous. Each identical twin has his or
her own unique thoughts, talents, experiences,
and beliefs. Mr. Rifkin must learn that human
beings are more than just their DNA; they are
(75) the products of the continual and inscrutably
complex interactions of environment and biology.
Human clones would be no different.
“But you are playing God!” we hear. It is the
cry of all whose power is threatened by the march
(80) of human progress. It is the reasoning of the Dark
Ages, used to keep the subservient masses in their
place. Every great step humanity has ever taken
has disrupted the "natural order." Should we be
shivering in caves, eating uncooked bugs, and
(85) dying of parasites, as nature intended?
But perhaps procreation is different—more
sacred. Then why have the technologies of fertility
enhancement, in vitro fertilization, embryo
transfer, and birth control become so widely
(90) accepted? Each of these technologies was met at
first with legions of strident opponents. But over
time, reality and compassion overcame unreason
and paranoia. Familiarity dissipates fear.
These supposedly "moral" objections are
(95) in fact impeding moral progress. With genetic
engineering, cloning, and stem cell research,
scientists finally have within their grasp
technologies that can provide ample food for
a starving world, cure devastating illnesses,
(100) and replace diseased organs. Only ignorant
superstition stands in their way.

Q. Passage 2 refers to the Twin’s Days Festival in line 68 as an example of

Detailed Solution for OneTime: Digital SAT Mock Test - 5 - Question 30

The author of Passage 2 mentions the Twins Days Festival (line 68) in order to demonstrate the absurdity of Jeremy Rifkin's statement that creating a genetic Xerox of a person is a horrendous crime (line 64). To the author of Passage 2, then, the Twins Days Festival represents the innocuousness (harmlessness) of genetic duplication, since twins are genetic duplicates, and nothing to be feared.

View more questions
8 docs|22 tests
Information about OneTime: Digital SAT Mock Test - 5 Page
In this test you can find the Exam questions for OneTime: Digital SAT Mock Test - 5 solved & explained in the simplest way possible. Besides giving Questions and answers for OneTime: Digital SAT Mock Test - 5, EduRev gives you an ample number of Online tests for practice

Top Courses for SAT

Download as PDF

Top Courses for SAT